You are on page 1of 83

1st CaMO 1969

Problem 1
a, b, c, d, e, f are reals such that a/b = c/d = e/f; p, q, r are reals, not all zero; and n is a positive
integer. Show that (a/b)n = (p an + q cn + r en)/(p bn + q dn + r fn ).

Solution
If a/b = c/d, then an/bn = cn/dn. So it is sufficient to show that if A/B = C/D then (pA + qC)/(pB
+ qD) = A/B, which is trivially true. [Apply it once with A = an, B = bn, C = cn, D = dn, then
again with p = 1.]

Problem 2

If x is a real number not less than 1, which is larger: √(x+1) - √x or √x - √(x-1)?

Solution

Answer: √x - √(x-1) is larger.

4x2 - 4x + 1 > 4x2 - 4x. Taking the positive square root of each side gives 2x - 1> 2√(x2-x).
Hence x2 + x > x2 - x + 1 + 2√(x2-x). Taking the positive square root, we get √(x2+x) > √(x2-x)
+ 1. Hence x + (x - 1) - 2√(x2-x) > (x + 1) + x - 2√(x2+x). Taking the positive square root, √x -
√(x-1) > √(x+1) - √x.

Problem 3

A right-angled triangle has longest side c and other side lengths a and b. Show that a + b ≤
c√2. When do we have equality?

Solution

Answer: equality iff a = b.

We have 2a2 + 2b2 = 2c2 (Pythagoras). Also a2 - 2ab + b2 = (a - b)2 >= 0, with equality iff a =
b. Hence a2 + 2ab + b2 <= 2c2 with equality iff a = b.

Problem 4
The sum of the distances from a point inside an equilateral triangle of perimeter length p to
the sides of the triangle is s. Show that s √12 = p.

Solution

Let the triangle be ABC and the point inside be X. Let the perpendiculars from X to BC, CA,
AB be XD, XE, XF respectively. The area of triangle ABX = base x height/2 = AB·XD/2.
Similarly for BCX and CAX. But area ABX + area BCX + area CAX = area ABC. Hence
AB·XD/2 + BC·XE/2 + CA·XF/2 = AB2(√3)/4. So s = AB (√3)/2 = p (√3)/6 = p/√12.

Problem 5

ABC is a triangle with |BC| = a, |CA| = b. Show that the length of the angle bisector of C is
(2ab cos C/2)/(a + b).

Solution

Let the bisector be CD. Then the length of the perpendicular from C to BC is CD sin C/2. The
perpendicular from C to AC has the same length. Hence area DBC = (a CD sin C/2)/2 and
area DAC = (b CD sin C/2)/2. But area DBC + area DAC = area ABC = (ab sin C)/2 = ab sin
C/2 cos C/2. Hence CD = (2ab cos C/2)/(a + b).

Problem 6

Find 1.1! + 2.2! + ... + n.n! .

Solution

We show by induction that 1·1! + 2·2! + ... + n·n! = (n+1)! - 1. It is obviously true for n = 1.
Suppose it is true for n, then 1.1! + ... + (n+1) (n+1)! = (n+1)! - 1 + (n+1) (n+1)! = (n+2)! - 1,
so it is true for n+1.

Problem 7

Show that there are no integer solutions to a2 + b2 = 8c + 6.


Solution

Squares are always congruent to 0, 1 or 4 mod 8. So a2 + b2 must be congruent to 0, 1, 2, 4, or


5 mod 8. But 8c + 6 is congruent to 6 mod 8.

Problem 8

f is a function defined on the positive integers with integer values. Given that (1) f(2) = 2, (2)
f(mn) = f(m) f(n) for all m,n, and (3) f(m) > f(n) for all m, n such that m > n, show that f(n) =
n for all n.

Solution

We show by induction that f(m) = m for all m such that 2n-1n <= m <= 2n.

We have f(2.1) = f(2) f(1), so f(1) = 1, so the result is true for n = 1. Suppose it is true for n.
Then f(2n) = 2n. Hence f(2n+1) = f(2n2n) = f(2n) f(2n) = 2n2n = 2n+1. But there are 2n - 1 values
strictly between 2n and 2n+1. So, by (3), the result is true for n+1.

Problem 9

Show that the shortest side of a cyclic quadrilateral with circumradius 1 is at most √2.

Solution

Let the quadrilateral be ABCD and let the circumcenter be O. We have AB = 2 sin AOB/2.
Now ∠AOB + ∠BOC + ∠COD + ∠DOA = 360o. So the smallest of the angles AOB/2,
BOC/2, COD/2, DOA/2 is not greater than 45o. But sin x is increasing over the range 0 to 45o,
so the shortest side has length at most 2 sin 45o = √2.

Problem 10

P is a point on the hypoteneuse of an isosceles, right-angled triangle. Lines are drawn through
P parallel to the other two sides, dividing the triangle into two smaller triangles and a
rectangle. Show that the area of one of these component figures is at least 4/9 of the area of
the original triangle.

Solution
Let the triangle be ABC with hypoteneuse BC. Let the feet of the perpendiculars from P to
AB and AC by X, Y respectively. If BP >= 2/3 BC, then area XBP is at least 4/9 area ABC.
Similarly, if BP < 1/3 BC, then area YPC is at least 4/9 area ABC. The sides of the rectangle
are equal to BP and BC, so its area is BP(BC - BP) = BC2/4 - (BP - BC/2)2. Hence if BP lies
between BC/3 and 2BC/3, then the area of the rectangle is at least BC2(1/4 - 1/36) = 2/9 BC2 =
4/9 area ABC.

2nd CaMO 1970

Problem 1

Find all triples of real numbers such that the product of any two of the numbers plus the third
is 2.

Solution

Answer: (1, 1, 1) and (-2, -2, -2).

Let the numbers by x, y, z, so x + yz = 2 (1), y + zx = 2 (2), z + xy = 2 (3). Subtracting (2)


from (1) gives (x - y)(1 - z) = 0, so x = y or z = 1. If x = y, then x + xz = 2 and z + x2 = 2, so x3
- 3x + 2 = 0. Factorising, (x - 1)2(x + 2) = 0, so x = 1, y = 1, z = 1 or x = -2, y = -2, z = -2. If z
= 1, then x + y = 2 and xy = 1. Hence x2 - 2x + 1 = 0. Factorising (x - 1)2 = 0, so x = 1.

Problem 2

The triangle ABC has angle A > 90o. The altitude from A is AD and the altitude from B is BE.
Show that BC + AD ≥ AC + BE. When do we have equality?

Solution

This inequality is awkward because it is weak. In fact BC + AD ≥ AC + AB.

We have BC2 = AB2 + AC2 - 2AB.AC cos A, 2AD·BC = 2 area ABC = AB·AC sin A, so (BC
+ AD)2 ≥ BC2 + 2AD·BC ≥ AB2 + AC2 + 2AB·AC (sin A - cos A). So it is sufficient to show
that sin A - cos A ≥ 1. Put A = x + 3π/4, where -π/4 ≤ x <= π/4. Then sin A - cos A = sin x cos
3π/4 + cos x sin 3π/4 - cos x cos 3π/4 + sin x sin 3π/4 = √2 cos x ≥ 1 with equality iff x = ±π/4
or A = π/2 or π. Thus BC + AD ≥ AB + AC with equality iff AD = 0 and ∠A = π, in other
words iff B lies on the ray CA. Then AB ≥ BE with equality iff AE = 0, so BC + AD ≥ AC +
BE with equality iff B = A.
Problem 3

Every ball in a collection is one of two colors and one of two weights. There is at least one of
each color and at least one of each weight. Show that there are two balls with different color
and different weight.

Solution

Let the colors be red and blue and the weights 1 and 2. A ball can be R1, R2, B1 or B2 (with
the obvious notation). If the result is false, then we cannot have R1 and B2 and we cannot
have R2 and B1. That means all balls must be R1 and R2, or R1 and B1, or R2 and B2, or B1
and B2. But in the first case there is no blue ball, in the second case no weight 2, in the third
case no weight 1, and in the last case no red ball.

Problem 4

Find all positive integers whose first digit is 6 and such that the effect of deleting the first
digit is to divide the number by 25. Show that there is no positive integer such that the
deletion of its first digit divides it by 35.

Solution

Let the smaller number be N and have n digits. Then the larger number is N + 6.10n. So 25N =
N + 6.10n, so 4N = 10n and N = 250...0. It is easily checked that 6250...0 (625 or 625 followed
by one or more zeros) work.

In the second case we would have 35N = N + k.10n, where k is one of 1, 2, ... , 9. But then
17N = k·10n, which is not possible because the rhs is not divisible by 17.

Problem 5

A quadrilateral has one vertex on each side of a square side 1. Show that the sum of the
squares of its sides is at least 2 and at most 4.

Solution

Let the quadrilateral be ABCD. Suppose A is a distance w, 1-w from the two nearest vertices
of the square. Define x, y, z similarly. Then the sum of the squares of the sides of the
quadrilateral is w2 + (1-w)2 + x2 + (1-x)2 + y2 + (1-y)2 + z2 + (1-z)2. But w2 + (1-w)2 = 2(w -
1/2)2 + 1/2 which is at least 1/2 and at most 1. Similarly for the other pairs of terms.

Problem 6

Given three non-collinear points O, A, B show how to construct a circle center O such that the
tangents from A and B are parallel.

Solution

Take D so that AOBD is a parallelogram. Let the line perpendicular to DO through O meet
the circles on diameters AO and BO (again) at P and Q respectively. Then the required circle
has radius OP.

Obviously PQ is perpendicular to AP and BQ, so it is sufficient to show that OP = OQ. Now


∠OPA = 90o (AO diameter of circle OPA), and ∠POD = 90o (by construction), so PA is
parallel to OD, so OP = perpendicular distance of A from line OD. Similarly OQ =
perpendicular distance of B from line OD. But the two distances are the same since AOBD is
a parallelogram. Hence OP = OQ.

Problem 7

Given any sequence of five integers, show that three terms have sum divisible by 3.

Solution

We can replace each integer by its remainder on division by 3. If three remainders are the
same, then we can take those three integers. Similarly, if three remainders are different, then
we can take those three integers. But one or the other must be true.

Problem 8

P lies on the line y = x and Q lies on the line y = 2x. Find the equation for the locus for the
midpoint of PQ, if |PQ| = 4.

Solution
Let P be (a, a), Q be (b, 2b). Then the midpoint is ( (a+b)/2, (a+2b)/2) = (x, y). We have (a -
b)2 + (a - 2b)2 = 16, so 2a2 - 6ab + 5b2 = 16. We want to recast this as an equation connecting
x and y. We have a = 2(2x - y), b = 2(y - x), so 8(2x - y)2 - 24(2x - y)(y - x) + 20(y - x)2 = 15,
or 2(4x2 - 4xy + y2) + 6(2x2 - 3xy + y2) + 5(x2 - 2xy + y2) = 4. So 25x2 -36xy + 13y2 = 4. The
locus is a long, thin ellipse centered on the origin.

Problem 9

Let a1 = 0, a2n+1 = a2n = n. Let s(n) = a1 + a2 + ... + an. Find a formula for s(n) and show that s(m
+ n) = mn + s(m - n) for m > n.

Solution

We have 1 + 2 + ... + n = n(n+1)/2, so s(2n+1) = n(n+1), s(2n) = s(2n+1) - n = n2. We may


combine these into a single formula: s(n) = [n2/4].

m and n have the same parity. If they are both even, then s(m+n) - s(m-n) = (m+n)2/4 - (m-
n)2/4 = mn. If they are both odd, then s(m+n) = (m+n)2/4 - 1/4, s(m-n) = (m-n)2/4 - 1/4 and
s(m+n) - s(m-n) = mn.

Problem 10

A monic polynomial p(x) with integer coefficients takes the value 5 at four distinct integer
values of x. Show that it does not take the value 8 at any integer value of x.

Solution

We have p(x) - 5 = (x-a)(x-b)(x-c)(x-d) q(x) for some polynomial q(x) with integer
coefficients and some integers a, b, c, d. Now suppose x is an integer. If x equals a, b, c, or d,
then p(x) = 5, not 8. If x is not equal to a, b, c or d, then each of x-a, x-b, x-c, x-d is a distinct
integer. So the smallest possible absolute value for the product (x-a)(x-b)(x-c)(x-d) is |1|.|-1|.|
2|.|-2| = 4. The smallest possible value for |q(x)| is 1, so |p(x) - 5| is at least 4. So p(x) cannot
take the value 8 (if x is an integer).

3rd CaMO 1971


Problem 1
A diameter and a chord of a circle intersect at a point inside the circle. The two parts of the
chord are length 3 and 5 and one part of the diameter is length 1. What is the radius the circle?

Solution
Let the chord be AB, the diameter be CD and the point of intersection X. Then XA.XB =
XC.XD, so the other part of the diameter has length 15. So the diameter has length 16 and the
radius is 8.

Problem 2

If two positive real numbers x and y have sum 1, show that (1 + 1/x)(1 + 1/y) ≥ 9.

Solution

8(x - 1/2)2 ≥ 0, so 8x2 - 8x + 2 ≥ 0, so -x2 + x + 2 ≥ 9x - 9x2, or (x+1)(2-x) ≥ 9x(1-x). Putting y


= 1-x, this becomes (x+1)(1+y) ≥ 9xy. Since x and y are positive, we may divide by xy to get
the result.

Problem 3

ABCD is a quadrilateral with AB = CD and angle ABC > angle BCD. Show that AC > BD.

Solution

We have AC2 = AB2 + BC2 - 2AB·BC cos ABC, BD2 = CD2 + BC2 - 2CD·BC cos BCD. But
AB = CD and cos ABC < cos BCD, so the result follows.

Problem 4

Find all real a such that x2 + ax + 1 = x2 + x + a = 0 for some real x.

Solution

Subtracting, (a - 1)(x - 1) = 0, so a = 1 or x = 1. If a = 1, then the two quadratics are the same


and have two complex roots. So we cannot have a = 1. If a = -2, the first quadratic has roots 1,
1 and the second has roots 1, -2. So a = -2 is the unique solution.
Problem 5

A polynomial with integral coefficients has odd integer values at 0 and 1. Show that it has no
integral roots.

Solution

Let the polynomial be p(x). If x is an even integer, then every term of p(x) involving x is an
even integer, so p(x) must have the same parity as p(0), which is odd, so it cannot be zero. If x
is an odd integer, then every term of p(x) has the same parity as the corresponding term of
p(1). Hence p(x) has the same parity as p(1), so it must be odd and hence cannot be zero.

Problem 6

Show that n2 + 2n + 12 is not a multiple of 121 for any integer n.

Solution

It is easy to check that n(n+2) + 1 = 1, 3, 9, 5, 3, 3, 5, 9, 4, 1, 0 mod 11 for n = 0, 1, 2, 3, 4, 5,


6, 7, 8, 9, 10 mod 11. So if n2 + 2n + 12 is divisible by 11, then we must have n = -1 mod 11.
But in that case n = 11k - 1 for some k, and so n2 + 2n + 12 = 121k2 - 1 + 12 = 11 mod 121. So
we can never have n2 + 2n + 12 a multiple of 121.

Problem 7

Find all five digit numbers such that the number formed by deleting the middle digit divides
the original number.

Solution

Let the number be n = AhB, in other words 1000A + 100h + B, with 10 ≤ A < 100, 0 ≤ h ≤ 10,
0 ≤ B < 100. Then we have 1000A + 100h + B = k(100A + B) for some integer k. If k > 10,
then k(100A + B) - 1000A ≥ 100A + B > 100h + B, which is not possible. If k < 10, then
1000A + 100h + B - k.100A ≥ 100A + B ≥ 1000 > kB, which is not possible. So we must
have k = 10. Hence 100h = 9B. But the that implies that 9 divides h, so h = 0 or 9. It cannot be
9, because 9B < 900, so the only solution is h = B = 0. Hence n = 10000, 11000, 12000, ... , or
99000. [It is obvious that they are solutions.]
Problem 8

Show that the sum of the lengths of the perpendiculars from a point inside a regular pentagon
to the sides (or their extensions) is constant. Find an expression for it in terms of the
circumradius.

Solution

Use vectors. Let the pentagon be ABCDE. Let the point be K. Take an origin O and let the
vectors OA, OB, OC, OD, OE, OK be a, b, c, d, e, k respectively. Then (a - k) x (a - b) = is a
vector normal to the plane of the pentagon and with length AB times the length of the
perpendicular from K to AB. Hence (a - k) x (a - b) + (b - k) x (b - c) + (c - k) x (c - d) + (d -
k) x (d - e) + (e - k) x (e - a) (*) is a vector normal to the plane of the pentagon with length
AB times the sum of the lengths of the perpendiculars from K to the sides. But after
expansion the terms involving k cancel in pairs and hence sum to zero. So the sum of the
perpendicular lengths is independent of K. [Note that for K inside the pentagon all five cross
products vectors in (*) point in the same direction, but for K outside the pentagon, some of
them point in opposite direction, so the argument no longer holds.]

Problem 9

Find the locus of all points in the plane from which two flagpoles appear equally tall. The
poles are heights h and k and are a distance 2a apart.

Solution

Let the flagpole height h be at H, and the flagpole height k be at K. Let A point on the
segment HK such that AH/AK = h/k and let B be the other point on the line HK such that
BH/BK = h/k. Then the locus is the circle with diameter AB. If h = k, then there is no point B
and the locus is the perpendicular bisector of AB.

This is a well-known result (the circle of Apollonius) which scarcely needs proving. The case
h = k is obvious. So assume h > k. Suppose P is such that HP/PK = h/k. Let the internal and
external bisectors of ∠HPK meet the line HK at A' and B' respectively. Let the line through K
parallel to PB' meet the line HP at X. Then HA'/A'K = HP/PK = h/k and HB'/KB' = HP/XP
(HPB' and HXK similar) = HP/PK = h/k. So A' = A and B' = B. But ∠A'PB' = 90 o, so P lies
on the circle diameter AB. Conversely, if P lies on that circle, then let the line through K
parallel to PB meet PH at X, and let the line through K parallel to PA meet the line PH at Y.
Then HP/PY = HA/AK = h/k and HP/PX = HB/KB = h/k. So PX = PY. But ∠XKY = 90 o, so P
is the center of the circle XKY, so PY = PK. Hence HP/KP = HP/PY = h/k, so P is part of the
locus.
Problem 10

n people each have exactly one unique secret. How many phone calls are needed so that each
person knows all n secrets? You should assume that in each phone call the caller tells the
other person every secret he knows, but learns nothing from the person he calls.

Solution

Everyone must receive a call after n-2 calls have been made. For calls up to and including the
(n-2)th can communicate in total at most n-2 secrets, which is less than the n-1 which each
person needs to receive. Thus the minimum number of calls is 2n-2. This can be achieved as
follows. Let the people be A1, A2, ... , An. A1 calls A2, then A2 calls A3, and so on up to An-1
calls An. After that call An knows all the secrets. He then calls everyone else.

4th CaMO 1972

Problem 1
Three unit circles are arranged so that each touches the other two. Find the radii of the two
circles which touch all three.

Solution
The two circles have center at the center of the triangle, which is the centroid and hence a
distance 2/3 √3 from the center of each of the unit circles. The two required radii are therefore
2/√3 ± 1.

Problem 2

x1, x2, ... , xn are non-negative reals. Let s = ∑i<j xixj. Show that at least one of the xi has square
not exceeding 2s/(n2 - n).

Solution

If not then each term of s exceeds 2s/(n2-n). But there are n(n-1)/2 terms.

Problem 3
Show that 10201 is composite in base n > 2. Show that 10101 is composite in any base.

Solution

Let the base be b, then 10201 = (b4 + 2b2 + 1) = (b2 + 1)2 = 1012. 10101 = (b4 + 2b2 + 1) - b2 =
(b2 + b + 1)(b2 - b + 1). We have b > 1, so each factor is > 1.

Problem 4

Show how to construct a convex quadrilateral ABCD given the lengths of each side and the
fact that AB is parallel to CD.

Solution

If AB = CD, then the quadrilateral is a parallelogram, so we must have BC = DA. In this case
the construction is not unique. Draw AB, then any C on the circle center B radius BC. Then
take the line through C parallel to AB to get D.

If AB > CD, then first construct a triangle ADX with sides AX = AB-CD and DX = BC. Then
take B on the ray AX the required distance from A.

Problem 5

Show that there are no positive integers m, n such that m3 + 113 = n3.

Solution

We have n3 = m3 mod 11. But we can easily check that 03, 13, 23, ... , 103 = 0, 10, 3, 6, 2, 7, 4,
9, 5, 8, 1 mod 11, so we must have n = m mod 11. Obviously we cannot have n = m, so we
must have n = m+11k. But (m+11)3 - m3 = 3·11k·m2 + 3·(11k)2m + (11k)3 > (11k)3 ≥ 113. So
there are no solutions.

Problem 6

Given any distinct real numbers x, y, show that we can find integers m, n such that mx + ny >
0 and nx + my < 0.
Solution

If x > y, take m = 1, n = -1, then mx + ny = x - y > 0 and nx + my = y - x < 0. Conversely, if x


< y, take m = -1, n = 1, then mx + ny = y - x > 0, nx + my = x - y < 0.

Problem 7

Show that the roots of x2 - 198x + 1 lie between 1/198 and 197.9949494949... . Hence show
that √2 < 1.41421356 (where the digits 421356 repeat). Is it true that √2 < 1.41421356?

Solution

Let f(x) = x2 - 198x + 1. Then f(1/198) = f(198 - 1/198) = 1/1982. But f(x) = (x-99)2 - 9800, so
f(x) is decreasing for x < 99 and increasing for x > 99. Hence f(x) > 0 for x < 1/198 and x >
198 - 1/198 = 197.9949494949... .

The roots are x = 99 ± √9800 = 99 ± 70√2. Hence √2 < (99 - 1/198)/70 = 19601/13860 .(*)

1.41421356 = 1 + 41/100 + 421356/108 (1 + 1/106 + 1/1012 + ... ) = 1 + 41/100 +


421356/(108(1 - 1/106) ) = 1 + 41/100 + 421356/99999900 = 19601/13860.

There may be a neater way, but multiplying out by hand to get the square is not that hard:
1.999 999 993 287 873 6 < 2. So 1.41421356 < √2.

Problem 8

X is a set with n elements. Show that we cannot find more than 2n-1 subsets of X such that
every pair of subsets has non-empty intersection.

Solution

There are 2n subsets. But they form 2n-1 pairs: A, X-A and we can only have one member of
each pair.

Problem 9

Given two pairs of parallel lines, find the locus of the point the sum of whose distances from
the four lines is constant.
Solution

This is slightly messy. Suppose first the lines are all parallel. Suppose the distance between
adjacent lines are a, b, c. Then any point between the inner two lines has the same sum a + 2b
+ c. For d < a + 2b + c, there are no points with sum d. For d > a + 2b + c, there is a line of
points.

Now suppose the lines are not all parallel. Their points of intersection form a parallelogram
ABCD. Suppose the distances between the opposite sides are a and b. Then any point inside
the parallelogram has sum a + b. Clearly there are no points with smaller sum. For larger
sums the locus is an octagon with opposite sides parallel and vertices on the parallel lines.
Alternate sides lie inside a pair of parallel lines and are parallel to the other pair of parallel
lines. A side not inside a pair of parallel lines forms an isosceles triangle with the point of
intersection of the two lines containing its vertices.

[Note that all points on the base AB of an isosceles triangle C have the same sum of distances
to the sides AC and BC. If the point X is on the base, then the sum is XA sin XAC + XB sin
XBC = (XA + XB) sin XAC = AB sin A.]

Problem 10

Find the longest possible geometric progression in {100, 101, 102, ... , 1000}.

Solution

128, 192, 288, 432, 648, 972 has six terms and ratio 3/2. We show it is the best possible.

The ratio must be rational. Suppose it is h/k. If there are more than 6 terms, then the first term
must be divisible by k6 and the last term must be divisible by h6. If k >= 3, then h > k, so h >=
4. But 46 = 4096 > 1000, so the last term is not divisible by h6. So k < 3. We cannot have k =
1, because then the last term is at least h6 times the first term, which is impossible. So k = 2, h
= 3. So the first term must be a multiple of 64. The smallest such is 128. But then we only get
6 terms.

5th CaMO 1973


Problem 1
(1) For what x do we have x < 0 and x < 1/(4x) ? (2) What is the greatest integer n such that
4n + 13 < 0 and n(n+3) > 16? (3) Give an example of a rational number between 11/24 and
6/13. (4) Express 100000 as a product of two integers which are not divisible by 10. (5) Find
1/log236 + 1/log336.

Solution
(1) x < -2. (2) n = -6. (3) 17/37. (4) 32 x 3125. (5) Let k = log23, then 1/k = log32. We have
log236 = log24 + log29 = 2 + 2k, log336 = log39 + log34 = 2 + 2/k = 2(1+k)/k. So 1/log236 +
1/log336 = 1/(2(1+k)) + k/(2(1+k)) = 1/2.

Problem 2

Find all real numbers x such that x + 1 = |x + 3| - |x - 1|.

Solution

If x ≥ 1, then x+1 = x+3 - (x-1) = 4, so x = 3. If -3 ≤ x < 1, then x+1 = (x+3) - (1-x) = 2x+2, so
x = -1. If x < -3, then x+1 = -(x+3) - (1-x) = -4, so x = -5.

Problem 3

Show that if p and p+2 are primes then p = 3 or 6 divides p+1.

Solution

One of p, p+1, p+2 must be a multiple of 3, for p > 3, p and p+2 cannot be, so p+2 must be a
multiple of 3. Similarly, p+1 or p+2 must be even, p+1 cannot be, so p+1 must be even.

Problem 4

Let P0, P1, ... , P8 be a convex 9-gon. Draw the diagonals P0P3, P0P6, P0P7, P1P3, P4P6, thus
dividing the 9-gon into seven triangles. How many ways can we label these triangles from 1
to 7, so that Pn belongs to triangle n for n = 1, 2, ... , 7.

Solution

P0P7P8 must be 7. Hence P0P6P7 must be 6. Hence P0P3P6 must be 3. Hence P0P1P3 must be 1.
Hence P1P2P3 must be 2. Also P3P4P6 must be 4, so P4P5P6 must be 5. So there is only one way.
Problem 5

Let s(n) = 1 + 1/2 + 1/3 + ... + 1/n. Show that s(1) + s(2) + ... + s(n-1) = n s(n) - n.

Solution

Induction. Obviously true for n = 1 (taking lhs to be empty) and n = 2. Suppose it is true for n.
Then s(1) + s(2) + ... + s(n) = n s(n) - n + s(n) = (n+1) s(n) - n = (n+1)s(n+1) - (n+1).

Problem 6

C is a circle with chord AB (not a diameter). XY is any diameter. Find the locus of the
intersection of the lines AX and BY.

Solution

Answer: the orthogonal circle through A and B.

Let AX and BY intersect at P. Suppose X and Y are both on the major arc AB. ∠BAY +
∠ABY = 180o - ∠AYB. So ∠ABY + ∠BAX = 270o - ∠AYB. Hence ∠PAB + ∠PBA = 90
+ ∠AYB, so ∠APB = 90 - ∠AYB. But ∠AYB is constant, so ∠APB is constant and hence P
lies on a circle through A and B. If we take the limit as X tends to A, then AY becomes a
diameter, so ∠ABY becomes 90o, so AP becomes a diameter. But ∠YAP = 180o - ∠APB -
∠AYB = 90o, so the circles are orthogonal.

A similar argument works if we take Y on the major arc, but X on the minor arc. So any point
on the locus must belong to the orthogonal circle.

Let C have center O and the orthogonal circle through A and B have center O'. Let the lines
PA and PB meet C again at X and Y respectively. We show that XY is a diameter (so that P
belongs to the locus).

If P is any point on the minor arc AB, the angle between BY and the tangent OB equals
∠PAB. The angle betwen BX and the tangent O'B equals ∠BAX. But PAB and BAX are the
same angle and OB and O'B are perpendicular, so BX and BY are perpendicular. Hence XY is
a diameter.

If P is any point on the major arc AB, then the angle between BY and the tangent O'B equals
angle BAY. The angle between BP and the tangent OB equals ∠BAP. But these two angles
sum to 90o, so ∠BAY + ∠BAP = 90o. Hence AY is perpendicular to AX, so XY is a
diameter.
Problem 7

Let an = 1/(n(n+1) ). (1) Show that 1/n = 1/(n+1) + an. (2) Show that for any integer n > 1 there
are positive integers r < s such that 1/n = ar + ar+1 + ... + as.

Solution

(1) 1/(n+1) + an = (n + 1)/(n(n+1) ) = 1/n. (2) We have ar + ar+1 + ... + as = 1/r - 1/(r+1) +
1/(r+1) - 1/(r+2) + ... + 1/s - 1/(s+1) = 1/r - 1/(s+1). Take r = n-1 and s = n(n-1) - 1. Then 1/r -
1/(s+1) = 1/(n-1) - 1/(n(n-1) ) = 1/n.

6th CanMO 1974

Problem 1
(1) given x = (1 + 1/n)n, y = (1 + 1/n)n+1, show that xy = yx. (2) Show that 12 - 22 + 32 - 42 + ... +
(-1)n+1n2 = (-1)n+1(1 + 2 + ... + n).

Solution
(1) Put k = (1 + 1/n). Then n kn+1 = (n+1) kn or ny = (n+1)x. Hence kny = k(n+1)x. Hence xy = yx.
(2) Induction. Obvious for n = 1. Suppose it is true or n. Then we need to show that (-1)n
(n+1)2 = (-1)n+2(1 + 2 + ... + n+1) - (-1)n+1(1 + 2 + ... + n). In other words, (n+1)2 = (1 + 2 + ...
+ n+1) + (1 + 2 + ... + n) (*). But 1 + 2 + ... + n = n(n+1)/2 (n/2 pairs of terms each with sum
n+1) and 1 + 2 + ... + n+1 = (n+1)(n+2)/2, so the rhs of (*) is (n + n+2)(n+1)/2 = (n+1)2.

Problem 2

Given the points A (0, 1), B (0, 0), C (1, 0), D (2, 0), E (3, 0), F (3, 1). Show that ∠FBE +
∠FCE = ∠FDE.

Solution

∠FDE = ∠FBE + ∠BFD, so it is sufficient to prove that ∠FCE = ∠BFD. But cos FCE =
2/√5. We have BD2 = BF2 + EF2 - 2BF.FE cos BFD, so cos BFD = 2/√5 also.

Problem 3
All coefficients of the polynomial p(x) are non-negative and none exceed p(0). If p(x) has
degree n, show that the coefficient of xn+1 in p(x)2 is at most p(1)2/2.

Solution

Let p(x) be anxn + an-1xn-1 + ... + a0. We are given that am ≤ a0 for all m. The coefficient of xn+1
in p(x)2 is k = a1an + a2an-1 + ... + ana1 ≤ a0(a1 + a2 + ... + an). But p(1)2 = sum of all terms aiaj.
All the terms are non-negative, so if we drop the terms not involving a0 we do not increase the
total and get p(0)2 ≥ 2a0(a1 + ... + an) ≥ 2k.

Problem 4

What is the maximum possible value for the sum of the absolute values of the differences
between each pair of n non-negative real numbers which do not exceed 1?

Solution

Let the numbers be x1 >= x2 >= ... >= xn. Then the sum is (x1 - x2) + (x1 - x3) + ... + (x1 - xn) +
(x2 - x3) + ... + (x2 - xn) + (x3 - x4) + ... + (x3 - xn) + ... + (xn-1 - xn) = (n-1) x1 + (n-2) x2 + (n-3)
x3 + ... + xn-1 - (n-1) xn - (n-2) xn-1 - ... - x2 = (n-1) x1 + (n-3) x2 + (n-5) x3 + ... + (n - (2n-1)) xn.
Since all xi are non-negative we maximise the sum by taking the xi with non-positive
coefficients to be zero and the xi with positive coefficients to be 1. That gives the maximum
M = (n-1) + (n-3) + (n-5) + ... . If n = 2m, then M = 1 + 3 + 5 + ... + 2m-1 = m2. If n = 2m+1,
then M = 2 + 4 + ... + 2m = m(m+1). We can combine that into a single formula as M = [n2/4].

Problem 5

AB is a diameter of a circle. X is a point on the circle other than the midpoint of the arc AB.
BX meets the tangent at A at P, and AX meets the tangent at B at Q. Show that the line PQ,
the tangent at X and the line AB are concurrent.

Solution

Let the lines AB and PQ meet at K. Let ∠XAB = x < 45o, and take units so that AB = 1. Then
QB = tan x, PA = cot x. So KB/(KB + 1) = KB/KA = QB/QA = tan2x, so KB = s2/(c2 - s2), KA
= c2/(c2 - s2), where s = sin x, c = sin x. Form a right-angled triangle with hypoteneuse KX by
dropping a perpendicular from X to AB. Then KX2 = (KA - c2)2 + (c s)2 = KA2s2 + KB2c2 =
s2c2/(c2 - s2)2 = KA·KB. Hence KX is the tangent at X.
Problem 6

What is the largest integer n which cannot be represented as 8a + 15b with a and b non-
negative integers?

Solution

Answer: 97 (=8.15 - 15 - 8).

98 = 6·15 + 8, 99 = 5·15 + 3·8, 100 = 4·15 + 5·8, 101 = 3·15 + 7·8, 102 = 2·15 + 9·8, 103 =
1·15 + 11·8, 104 = 13·8, 105 = 7·15. That gives 8 consecutive numbers. We can now get any
larger number by adding a multiple of 8 to one of 98-105. To see that 97 cannot be expressed
as 8a + 15b, note that b must be odd, but 97-15, 97-45, 97-75 are not multiples of 8.

For the general case of relatively prime positive m, n > 1, the largest number which cannot
be written as ma + nb is mn - m - n.

Problem 7

Bus A leaves the terminus every 20 minutes, it travels a distance 1 mile to a circular road of
length 10 miles and goes clockwise around the road, and then back along the same road to to
the terminus (a total distance of 12 miles). The journey takes 20 minutes and the bus travels at
constant speed. Having reached the terminus it immediately repeats the journey. Bus B does
the same except that it leaves the terminus 10 minutes after Bus A and travels the opposite
way round the circular road. The time taken to pick up or set down passengers is negligible. A
man wants to catch a bus a distance 0 < x < 12 miles from the terminus (along the route of
Bus A). Let f(x) the maximum time his journey can take (waiting time plus journey time to
the terminus). Find f(2) and f(4). Find the value of x for which f(x) is a maximum. Sketch
f(x).

Solution

The bus takes 20 mins for 12 miles or 5/3 min/mile. For x <= 1, the best strategy is to wait up
to 10 minutes for a returning bus, so f(x) = 10 + 5x/3. Similarly for 11 <= x <= 12, f(x) =
f(12-x).

For 1 < x <= 6, the worst case is that he just misses the right bus, so that the wrong bus comes
10 minutes later and the right bus 10 minutes after that. So he can wait 10 minutes and then
travel 12-x miles for a total time of 10 + 5(12 - x)/3 = 30 - 5x/3 or he can wait 20 minutes and
then travel x miles for a total time of 20 + 5x/3. The first is better for x >= 3. So,
summarising:

f(x) = 10 + 5x/3 for 0 ≤ x <= 1

20 + 5x/3 for 1 < x <= 3

30 - 5x/3 for 3 < x <= 6

10 + 5x/3 for 6 < x <= 9

40 - 5x/3 for 9 < x < 11

30 - 5x/3 for x ≥ 11

f(2) = f(4) = 23 1/3. The maximum value of 25 minutes occurs at x = 3 and x = 9.

7th CanMO 1975

Problem
Evaluate (1·2·4 + 2·4·8 + 3·6·12 + 4·8·16 + ... + n·2n·4n)1/3/(1·3·9 + 2·6·18 + 3·9·27 +
4·12·36 + ... + n·3n·9n)1/3.

Solution
The numerator cubed is 8(13 + 23 + ... + n3), the denominator cubed is 27(13 + 23 + ... + n3).
Hence the expression is 2/3.
Problem 2

Define the real sequence a1, a2, a3, ... by a1 = 1/2, n2an = a1 + a2 + ... + an. Evaluate an.

Solution

We show by induction that an = 1/(n(n+1) ). Obviously true for n = 1. Suppose it is true for n.
We need to show that (n+1)2/( (n+1)(n+2) ) - n2/( n(n+1) ) = 1/( (n+1)(n+2) ), or (n+1)/(n+2) -
n/(n+1) = 1/( (n+1)(n+2) ), or (n+1)2 - n(n+2) = 1, which is true.

Problem 3

Sketch the points in the x, y plane for which [x]2 + [y]2 = 4.

Solution

There are four squares with vertices at lattice points and centered at (2 1/2, 1/2), (1/2, 2 1/2),
(-1 1/2, 1/2), (1/2, -1 1/2). The first square includes all its interior points, the vertex (2, 0) (but
none of the others), all other points on the bottom and left-hand boundaries, but no points on
the top or right-hand boundaries. The others are similar.

Problem 4

Find all positive real x such that x - [x], [x], x form a geometric progression.

Solution

Let [x] = n. We have x - n < 1, so if n ≥ 2, then n/(x - n) > 2, but x/n < 2. We must have n ≥ 1,
otherwise n < x - n. Hence n = 1. Let x = 1 + y. Then 1/y = 1 + y, so y = (√5 - 1)/2 and x = (√5
+ 1)/2.

Problem 5
Four points on a circle divide it into four arcs. The four midpoints form a quadrilateral. Show
that its diagonals are perpendicular.

Solution

Let the circle have center O and the four points be A, B, C, D going around the circle counter-
clockwise. Measure angles at O counter-clockwise from A, so angle A = 0. Then the midpoint
of the arc AB has angle B/2 and the midpoint of the arc CD has angle C/2 + D/2, so the
midpoint of the chord joining them has angle (B + C + D)/4. The midpoint of the arc BC has
angle B/2 + C/2 and the midpoint of the arc DA has angle 180 + D/2. Hence the midpoint of
the chord joining them has angle 90 + (B + C + D)/4. So the two diagonals of the midpoint
quadrilateral are perpendicular.

Problem 6

15 guests with different names sit down at a circular table, not realizing that there is a name
card at each place. Everyone is in the wrong place. Show that the table can be rotated so that
at least two guests match their name cards. Give an example of an arrangement where just one
guest is correct, but rotating the table does not improve the situation.

Solution

Pigeon-hole principle. There are 15 possible positions for the table. Everyone must be correct
in one position. No one is correct in the initial position, so 15 people are correct in 14
positions, so at least two people must be correct in the same position.

Consider the arrangement:

card 1 2 3 4 5 6 7 8 9 10 11 12 13 14 15

person 1 9 2 10 3 11 4 12 5 13 6 14 7 15 8

A rotation of 1 brings 2 into line, a rotation of 3 brings 2 into line and so on. So no rotation
lines up two or more guests.

Problem 7

Is sin(x2) periodic?

Solution

No. The zeros are √(nπ), but these do not form an arithmetic progression.
Problem 8

Find all real polynomials p(x) such that p(p(x) ) = p(x)n for some positive integer n.

Solution

Let p(x) have highest term a xm. Then the highest term in p(p(x) ) is am+1xM, where M = m2,
and the highest term in p(x)n is anxmn. Hence m = n and a = 1. But that means p(p(x) ) = p(x)n +
the other terms, so the other terms must all be zero. Hence p(x) = xn.

8th CanMO 1976

Problem 1
Given four unequal weights in geometric progression, show how to find the heaviest weight
using a balance twice.

Solution
Let the weights be b, bd, bd2, bd3 with d > 1. Then d2(d - 1) > (d - 1), so bd3 + b > bd + bd2. So
if we test two weights against the other two then the heavier pair is sure to include the
heaviest single weight (bd3). The second weighing determines which weight in the pair is
heavier.

Problem 2

The real sequence x0, x1, x2, ... is defined by x0 = 1, x1 = 2, n(n+1) xn+1 = n(n-1) xn - (n-2) xn-1.
Find x0/x1 + x1x2 + ... + x50/x51.

Solution

We show by induction that xn = 1/n! for n ≥ 2. We have 1·2 x2 = 1·0. x1 - (-1) x0 = 1, so x2 =


1/2 and the result is true for n = 2. We also have 2·3 x3 = 2·1 x2 - 0. x1 = 1, so x3= 1/6 and the
result is true for n = 3.

Suppose it is true for n and n-1. Then n(n+1) xn+1 = n(n-1)/n! - (n-2)/(n-1)! = 1/(n-2)! - 1/( (n-
1) (n-3)! ) = (n-1 - (n-2) )/(n-1)! = 1/(n-1)!, so xn+1 = 1/(n+1)! and the result is true for n+1.
Thus x0/x1 + x1x2 + ... + x50/x51 = 1/2 + 2/(1/2) + (1/2)/(1/6) + ... + (1/50!)/(1/51!) = 1/2 + 4 + 3
+ 4 + 5 + ... + 51 = 3/2 + (1 + 2 + ... + 51) = 3/2 + 51.52/2 = 1327 1/2.

Problem 3

n+2 students played a tournament. Each pair played each other once. A player scored 1 for a
win, 1/2 for a draw and nil for a loss. Two students scored a total of 8 and the other players all
had equal total scores. Find n.

Solution

There are (n+2)(n+1)/2 matches, so the total score is (n+2)(n+1)/2. Let the other players score
k each. Then 8 + nk = (n+2)(n+1)/2, so n2 - (2k-3) - 14 = 0. We know this equation has one
root which is a positive integer. The product of the roots is -14, so the possibilities for the
roots are: 1, -14; 2, -7; 7, -2; 14, -1. Hence the sum of the roots is -13, -5, 5, or 13
(respecively). Hence k = -5, -1, 4 or 8 (respectively). But k must be non-negative, so n = 7 or
14 is a necessary condition.

We need to check that these values can be achieved. Take n = 7, so there are 9 players in total.
If every match is a draw, then every player draws 8 matches and scores 4, which satisfies the
conditions. Take n = 14, so there are 16 players in total. Suppose one player loses to
everyone, and all the other games end in a draw. Then the first player scores 0 and all the
other players score 1 + 14/2 = 8. That also satisfies the conditions.

Problem 4

C lies on the segment AB. P is a variable point on the circle with diameter AB. Q lies on the
line CP on the opposite side of C to P such that PC/CQ = AC/CB. Find the locus of Q.

Solution

If C is the midpoint of AB, then the locus is obviously the same circle. So assume C is closer
to A than B. Take B' on the line AB on the opposite side of A to B so that B'C = CB2/AC. We
show that the locus is the circle diameter B'B.

Let this circle have center O'. Then BB' = BC + B'C = BC(1 + BC/AC) = AB. BC/AC. Hence
B'O' = BO' = BO.BC/AC and O'C = BO' - BC = (BO/AC - 1)BC = OC.BC/AC. Now let Q' be
the intersection of this circle with the ray PC. Then the triangles OCP and O'CQ' are similar
(equal angle at C and OC/O'C = OP/O'Q). Hence PC/CQ = BC/AC. But there is only one
point on the ray PC with the required ratio, so Q' = Q.

Problem 5

Show that a positive integer is a sum of two or more consecutive positive integers iff it is not
a power of 2.

Solution

(m+1) + (m+2) + ... + n = (n - m)(n + m + 1)/2 = N. If it is a sum of two or more, then n - m


>= 2. If n - m = 2, then N = 2m + 3, which is odd. If n - m > 2, then N = a.b/2 where a and b
are two integers greater than 2 of opposite parity, so N must have an odd factor, so it cannot
be a power of 2.

Suppose N is not a power of 2. Then we can write 2N = ab with a, b of opposite parity and a,
b > 1. We may assume a > b. If a = n + m + 1, b = n - m, then n = (a + b - 1)/2, m = (a - b -
1)/2. Since a and b are of opposite parity, n and m are integers. m is non-negative and n is
positive. Also n - m ≥ 2. So we have N = (m+1) + (m+2) + ... + n as required.

[Examples: N = 12, then 2N = 3·8, so a = 8, b = 3, n = 5, m = 2 and 12 = 3 + 4 + 5; N = 24,


then 2N = 3·16, so a = 16, b = 3, n = 9, m = 6, 24 = 7 + 8 + 9.]

Problem 6

The four points A, B, C, D in space are such that the angles ABC, BCD, CDA, DAB are all
right angles. Show that the points are coplanar.

Solution

Using Pythagoras: (1) AC2 = AB2 + BC2, (2) BD2 = BC2 + CD2, (3) CA2 = CD2 + DA2, (4)
DB2 = DA2 + AB2. (1) + (3), (2) + (4) gives: 2 AC2 = 2 BD2, so AC = BD. So (1) and (2) give
AB = CD, and (2) and (3) give BC = DA.

So triangles CBD and ADB are congruent. So ∠CBD = ∠ADB = 90o - ∠ABD. So ∠CBD +
∠ABD = ∠CBA. Hence the line DB lies in the plane ABC, so the points are coplanar.

Problem 7
p(x, y) is a symmetric polynomial with the factor (x - y). Show that (x - y)2 is a factor.

Solution

We have p(x, y) = (x - y) q(x, y) for some polynomial q(x, y). But p(x, y) = p(y, x), so (x - y)
q(x, y) = (y - x) q(y, x), or q(y, x) = - q(x, y). Hence q(x, x) = 0, so q(x, y) has a factor (x - y).

Problem 8

A graph has 9 points and 36 edges. Each edge is colored red or blue. Every triangle has a red
edge. Show that there are four points with all edges between them red.

Solution

We show first that any 5 points must have a red triangle.

Comment. This is just the result that the Ramsey number R(3, 4) <= 9. To see that 9 is best
possible arrange 8 points in a circle and join each to its two neighbours and to the two points
adjacent to its neighbours. Then every three points have an edge joining two of them, but
there are no 4 points all joined. [Or if you prefer, color these edges red and join all other
pairs with blue edges.]

9th CanMO 1977

Problem 1
Show that there are no positive integers m, n such that 4m(m+1) = n(n+1).

Solution
If n > 2m, then n+1 ≥ 2m+2, so n(n+1) > 4m(m+1). If n < 2m, then n+1 <= 2m, so n(n+1) <
4m2. So we must have n = 2m, but then n(n+1) = 2m(2m+1) = 4m(m + 1/2) which does not
equal 4m(m+1).

Problem 2

X is a point inside a circle center O other than O. Which points P on the circle maximise angle
OPX?
Solution

Let the chord through X perpendicular to OX be AB. Extend AO to meet the circle again at C.
Then O is the midpoint of AC and X is the midpoint of AB, so OX = BC/2. Now let P be any
point on the major arc AB. Extend PO to meet the circle again ar R and extend PX to meet the
circle again at Q. Take S on the ray XQ so that XS = XP. Then as before OX = RS/2. But
∠RQS = 90o, so RS > RQ, so RQ < OX/2, so ∠OPX = ∠RPQ < ∠OAX. Thus A and B are
the points which maximise the angle.

Problem 3

Find the smallest positive integer b for which 7 + 7b + 7b2 is a fourth power.

Solution

Let N = 7 + 7b + 7b2. Then 7 divides N, so 73 = 343 must divide 1 + b + b2. 1 + 17 + 172 = 307
< 343, so b ≥ 18. In fact 1 + 18 + 182 = 343, so b = 18 is the smallest solution.

Problem 4

The product of two polynomials with integer coefficients has all its coefficients even, but at
least one not divisible by 4. Show that one of the two polynomials has all its coefficients even
and that the other has at least one odd coefficient.

Solution

Let the polynomials be anxn + ... + a1x + a0 and bmxm + ... + b1x + b0. Suppose each polynomial
has at least one odd coefficient. Then let ai be the odd coefficient with smallest i and bj be the
odd coefficient with smallest j. Then the coefficient of xi+j in the product is a sum of terms
ahbk with h + k = i + j. All terms except aibj have either h > i or k > j, so all terms except aibj
are even. Hence the sum is odd. Contradiction. So we may suppose that all ah are even. If all
the bk were also even, then every product term would be a multiple of 4, which is false, so at
least one bk is odd.

Problem 5
A right circular cone has base radius 1. The vertex is K. P is a point on the circumference of
the base. The distance KP is 3. A particle travels from P around the cone and back by the
shortest route. What is its minimum distance from K?

Solution

The key is to cut the cone along KP and unroll it to give the sector of a circle. If the sector is
bounded by KP and KP' and the arc PP', then evidently the particle travels along the straight
line PP'. It is closest to K when it is at the midpoint M. The arc PP' has length 2π, so the
∠PKP' is 2π/3. Hence ∠KPP' = π/6, so KM = 3/2.

Problem 6

The real sequence x1, x2, x3, ... is defined by x1 = 1 + k, xn+1 = 1/xn + k, where 0 < k < 1. Show
that every term exceeds 1.

Solution

We show by induction that 1 < xn < 1/(1 - k). It is true for n = 1. Suppose it is true for n. Then
xn+1 = 1/xn + k > 1 - k + k = 1 and xn+1 < 1 + k < 1/(1 - k).

Problem 7

Given m+1 equally spaced horizontal lines and n+1 equally spaced vertical lines forming a
rectangular grid with (m+1)(n+1) nodes. Let f(m, n) be the number of paths from one corner
to the opposite corner along the grid lines such that the path does not visit any node twice.
Show that f(m, n) ≤ 2mn.

Solution

Induction on n. If n = 0, then there is just one path, so f(m, 0) = 1 ≤ 2m. Suppose the result is
true for n. Let the (n+1)th line of nodes be A', B', ... , X' and the nth line of nodes be A, B, ... ,
X with A adjacent to A' etc. Suppose that A' is the destination. We now count the paths with a
given pattern of segments in the line A'X'. There are 2m such patterns (each of m segments can
be in or out). If the segments D'E'F' are in the pattern (but not C'D' or F'G'), then the path must
include DD'E'F'F. In the case of a subpath of segments ending in A', such as A'B'C', the path
must include A'B'C'C. In other words, we get a collection of subpaths each of which is three
or (in one case) two sides of a rectangle. If we replace these subpaths by the other sides of
these rectangles (DEF, A'ABC for the examples above), then we get a path for m, n.
Moreover, this mapping is obviously injective. So there are at most 2mn paths for any given
pattern and hence at most 2m2mn = 2m(n+1) paths in total.

10th CanMO 1978

Problem
A square has tens digit 7. What is the units digit?

Solution
Suppose the unsquared number has tens digit a and units digit b. The carry from b2 must be
odd (since 7 is odd), so b = 4 or 6. In either case the units digit of the square is 6. An example
is 262 = 576.

Problem 2

Find all positive integers m, n such that 2m2 = 3n3.

Solution

2 must divide n, so 8 must divide 2m2, so 2 must divide M. Similarly, 3 must divide m, so 9
divides 3n3 so 3 divides n. Hence 81 divides 2m2, so 9 divides m. Put m = 18M, n = 6N, then
648M2 = 648N3, so M2 = N3. Thus M is a cube, so finally we have for some integer k, m =
18k3 , n = 6k2.

Problem 3

Find the real solution x, y, z to x + y + z = 5, xy + yz + zx = 3 with the largest z.

Solution

We have 25 = (x + y + z)2 = (x2 + y2 + z2) + 2(xy + yz + zx) = (x2 + y2 + z2) + 6, so (x2 + y2 +


z2) = 19. Hence (x - 1/3)2 + (y - 1/3)2 + (z - 1/3)2 = (x2 + y2 + z2) -2(x + y + z)/3 + 1/3 = 19 -
10/3 + 1/3 = 16. So the maximum possible value of z is 4 1/3 (with solution x = 1/3, y = 1/3, z
= 4 1/3).

Problem 4
ABCD is a convex quadrilateral with area 1. The lines AD, BC meet at X. The midpoints of
the diagonals AC and BD are Y and Z. Find the area of the triangle XYZ.

Solution

Use vectors with origin X. Let XA = a, XB = b, XC = hb, XD = ka. Then area ABCD = area
XCD - area XAB = (hk - 1) |a x b| = 1, and area XYZ = |(a + hb) x (b + ka)/4| = (hk - 1)/4 |a
x b| = 1/4. ( b x a = - a x b, a x a = b x b = 0).

Problem 5

Two players play a game on an initially empty 3 x 3 board. Each player in turn places a black
or white piece on an unoccupied square of the board. Each player may play either color.
When the board is full player A gets one point for every row, column or main diagonal with 0
or 2 black pieces on it. Player B gets one point for every row, column or main diagonal with 1
or 3 black pieces on it. Can the game end in a draw? Which player has a winning strategy if
player A plays first? If player B plays first?

Solution

A draw is possible, for example:

W B W

B W B

W B W

If A plays first, A can play black in the center and thereafter the opposite color to B's last
move on the opposite side of the center (so if B plays in white in the middle of the top row,
then A returns black in the middle of the bottom row). This must give two black pieces on
each main diagonal and two black pieces on the central row and on the central column (a
score of 4 so far). Also two adjacent corners must be black and the other two white. If the
piece between the two adjacent corners is white, then A scores one for that row/col, if not A
scores one for the parallel line at the opposite side of the board (which must have no black
pieces). So A scores at least 5 and wins.
If B plays first, then B can win by a similar strategy. B plays white in the center and thereafter
the opposite color to A's last move on the opposite side of the center. A similar argument to
the above shows that this is a win for B.

Problem 6
Sketch the graph of x3 + xy + y3 = 3.

Solution

Close to the line y = -x far from the origin. A slight bulge away from the line (in the NE
direction into the first quadrant) near the origin. Passes through (0, k), (1, 1), (k, 0), where k is
31/3 = 1.44.

11th CanMO 1979

Problem 1
If a > b > c > d is an arithmetic progression of positive reals and a > h > k > d is a geometric
progression of positive reals, show that bc > hk.

Solution
Let a - d = 3t. Then bc = (d + 2t)(d + t) = d2 + 3td + 2t2. Also a/d = 1 + 3t/d, so hk = d2(1 +
3t/d) = d2 + 3td < bc.

Problem 2

Show that two tetrahedra do not necessarily have the same sum for their dihedral angles.

Solution

Take AB = BC = BD and AB normal to the plane BCD. Let M be the midpoint of CD. Then
the sum of the dihedral angles is 90o + 90o + ∠AMB + ∠CBD. So as CBD tends to a straight
line the sum tends to 180o + 90o + 180o = 450o. As CBD tends to zero the sum tends to 180o +
45o + 0o = 225o.

Problem 3

Given five distinct integers greater than one, show that the sum of the inverses of the four
lowest common multiples of the adjacent pairs is at most 15/16. [Two of the numbers are
adjacent if none of the others lies between.]

Solution
Let the numbers be a > b > c > d > e > 1. We need to show that 1/lcm(a, b) + 1/lcm(b, c) +
1/lcm(c, d) + 1/lcm(d, e) ≤ 15/16.

If the numbers are 2, 3, 4, 5, 6 then the sum is 1/6 + 1/12 + 1/20 + 1/30 = (10 + 5 + 3 + 2)/60
= 1/3 < 15/16. Otherwise we must have lcm(a, b) ≥ b ≥ 3, lcm(b, c) ≥ c ≥ 4, lcm(c, d) ≥ d ≥ 5,
lcm(d, e) ≥ e ≥ 7, so sum ≤ 1/3 + 1/4 + 1/5 + 1/7 = (140 + 105 + 84 + 60)/420 = 389/420 <
15/16.

11th CanMO 1979

Problem 4

A dog is standing at the center of a circular yard. A rabbit is at the edge. The rabbit runs round
the edge at constant speed v. The dog runs towards the rabbit at the same speed v, so that it
always remains on the line between the center and the rabbit. Show that it reaches the rabbit
when the rabbit has run one quarter of the way round.

Solution

Let the edge be the circle C. Let the center be O. Let AB be a diameter of C. Let S be the
midpoint of the arc AB. Let P be any point on the minor arc SB. Let C' be the circle on
diameter OB and let its center (the midpoint of OB) be O'. The rabbit runs from S towards B
around C. We show that the dog runs from O to B around the arc OB closer to the arc SB. Let
OP intersect C' at Q. Then angle QO'O = 2 x angle POS, so the distance along the circle C
from S to P equals the distance along the circle C' from O to Q.

Problem 5

The lattice is the set of points (x, y) in the plane with at least one coordinate integral. Let f(n)
be the number of walks of n steps along the lattice starting at the origin. Each step is of unit
length from one intersection point to another. We only count walks which do not visit any
point more than once. Find f(n) for n =1, 2, 3, 4 and show that 2n < f(n) ≤ 4·3n-1.

Solution

f(1) = 4 (any of 4 directions), f(2) = 12 (any of 3 directions for step 2), f(3) = 36 (any of 3
directions for step 3), f(4) = 100 (any of 3 for step 4, except for the 8 paths after step 3 which
go around 3 sides of a square - only 2 possible directions for each of those).
You cannot go back the way you came, so for steps 2 to n there is a choice of at most 3
directions. Hence f(n) <= 4.3n-1. If you always go N or E, then you can never visit a point
more than once, so f(n) > 2n.

12th CanMO 1980

Problem 1
If the 5-digit decimal number a679b is a multiple of 72 find a and b.

Solution
100 = 28 mod 72, 1000 = -8 mod 72, 10000 = -8 mod 72, so we require -8a - 48 + 196 + 90 +
b = 0 mod 72, or 22 - 8a + b = 0 mod 72. We have -72 < -50 = 22 - 8.9 + 0 ≤ 22 - 8a + b ≤ 22
+ 0 + 9 = 31 < 72, so 22 - 8a + b = 0, or 8a = 22 + b. So b = 2, a = 3. Check: 36792 = 511·72.

Problem 2

The numbers 1 to 50 are arranged in an arbitrary manner into 5 rows of 10 numbers each.
Then each row is rearranged so that it is in increasing order. Then each column is arranged so
that it is in increasing order. Are the rows necessarily still in increasing order?

Solution

Answer: yes.

After the row rearrangement, let the numbers be a1, a2, ... , a10 in the first row, b1, b2, ... , b10 in
the second row and so on.

Suppose that after the column rearrangement we have xi in column i of row k (where x is one
of a, b, c, d, e) and yj in column j of row k (where j > i and y is one of a, b, c, d, e). We wish to
show that xi < yj. There is one more numbers in column j in row k or above than there are in
column i above row k. So there must be some zj in column j and in row k or above with the
corresponding zi in column i in row k or below (z may equal x or y). So xi ≤ zi < zj ≤ yj.

Problem 3

Find the triangle with given angle A and given inradius r with the smallest perimeter.

Solution
It is fairly obvious that we take angle B = angle C. But we need to prove it.

The perimeter is 2r (cot A/2 + cot B/2 + cot C/2). Both 2r and cot A/2 are fixed, so we have to
minimise cot B/2 + cot C/2. Write B/2 = x + y, C/2 = x - y, where x = (B+C)/4, y = (B-C)/4.
So we have cot(x + y) + cot(x - y) = (1 - tan x tan y)/(tan x + tan y) + (1 + tan x tan y)/(tan x -
tan y) = (2 tan x + 2 tan x tan2y)/(tan2x - tan2y). Now tan x is fixed (because A and hence B+C
is fixed), so we both minimise the numerator and maximise the denominator by taking B = C.

Problem 4

A fair coin is tossed repeatedly. At each toss 1 is scored for a head and 2 for a tail. Show that
the probability that at some point the score is n is (2 + (-1/2)n)/3.

Solution

Induction on n. True for n = 1 because a score of 1 can only be achieved after the frist toss -
thereafter the score must exceed 1. Suppose the result is true for n.

If a score of n is achieved, then a score of n+1 can only be achieved by tossing a head on the
next throw. If a score of n is not achieved, then a score of n+1 must be achieved, because the
gap between scores can be at most 2. Hence the prob of achieving n+1 = (2 + (-1/2)n)/6 + (1 -
(2 + (-1/2)n)/3 = 2/3 + (1/3 - 1/6) (-1/2)n+1 = (2 + (-1/2)n+1)/3.

Problem 5

Do any polyhedra other than parallelepipeds have the property that all cross sections parallel
to any given face have the same perimeter?

Solution

Yes. The octahedron. Suppose it has edge length 1. You can cut along edges and fold the
faces flat, so that the six faces with a vertex in common with the top face form a 1 x 3
parallelogram. A plane parallel to the top face cuts this parallelogram in a line of length 3
parallel to the long sides.

13th CanMO 1981


Problem 1
Show that there are no real solutions to [x] + [2x] + [4x] + [8x] + [16x] + [32x] = 12345.

Solution
Let f(x) = [x] + [2x] + [4x] + [8x] + [16x] + [32x]. Obviously f is an increasing function (if x
< y, then f(x) ≤ f(y).). f(196) = 12348. But if x is just under 196, then [x], [2x], [4x], [8x],
[16x] and [32x] are all smaller by at least 1, so f(x) < 12342. Hence f never takes the value
12345.

Problem 2

The circle C has radius 1 and touches the line L at P. The point X lies on C and Y is the foot
of the perpendicular from X to L. Find the maximum possible value of area PXY (as X
varies).

Solution

Answer: (3√3)/8.

Let O be the center of C and QP a diameter. Let ∠QOX = x. Then PY = sin x and XY = 1 +
cos x, so the area is sin x(1 + cos x)/2. We claim that this has minimum value (3√3)/8. Put y =
sin x, so √(1 - y2) = cos x. We have (y - 3√3/4y)2 ≥ 0, so y2 - (3√3)/2y + 27/16y2 ≥ 0, or
27/16y2 - (3√3)/2y + 1 ≥ 1 - y2 (*). Now y ≤ 1, so (3√3)/4y > 1. Thus taking the positive
square roots of (*) we get: (3√3)/4y - 1 ≥ √(1 - y2) or y(1 + √(1 - y2)) ≤ (3√3)/4, as claimed.

Problem 3

Given a finite set of lines in the plane, show that an arbitrarily large circle can be drawn which
does not meet any of them. Show that there is a countable set of lines in the plane such that
any circle with positive radius meets at least one of them.

Solution

If all the lines are parallel, then there is an empty half-plane beyond the last line, in which we
can put arbitraril large circles. If not, take the convex hull of all the points of intersection.
Rays emerge from this hull and do not intersect outside it. Take two adjacent rays which are
not parallel, then they diverge, so arbitrarily large circles can be accomodated between them.

Take lines parallel to the coordinate axes through every rational point on the other axis.
Problem 4

p(x) and q(x) are real polynomials such that p(q(x) ) = q(p(x) ) and p(x) = q(x) has no real
solutions. Show that p(p(x) ) = q(q(x) ) has no real solutions.

Solution

Suppose p(p(a)) = q(q(a)) for some real a. Put b = p(a), c = q(a). So p(b) = q(c). Note that
since p(x) and q(x) are real polynomials, b and c are real. We cannot have b = c since p(x) =
q(x) has no real solutions. Now p(c) =p(q(a)) = q(p(a)) = q(b). Now put r(x) = p(x) - q(x), then
r(b) = p(b) - q(b). We cannot have r(b) = 0, since p(x) = q(x) has no real solutions. But r(b) =
p(b) - q(b) = q(c) - p(c) = - r(c), so r(x) changes sign between x = b and x = c, so it must have
a root between b and c. Contradiction. Hence p(p(x)) = q(q(x)) cannot have a real solution.

Problem 5

11 groups perform at a festival. Each day any groups not performing watch the others (but
groups performing that day do not watch the others). What is the smallest number of days for
which the festival can last if every group watches every other group at least once during the
festival?

Solution

Answer: 6 days.

It is not obvious how to approach this. Consider the case of n groups. It is easy to make do
with n days. For example, on day i, group i performs and the others watch. Or vice versa. For
n = 3, 3 days are necessary, 1 watching 2, 2 watching 3 and 3 watching 1 must occur on
separate days. It is not quite as obvious that 4 days are needed for n = 4. Suppose three days
suffice. Then on some day two groups perform. Wlog 1 and 2 perform on day 1. Then we
need two more days for 1 to watch 2 and 2 to watch 1. So assume 1 watches 2 on day 2 and 2
watches 1 on day 3. The only day 1 is in the audience is day 2, so 3 and 4 must perform on
day 2. Similarly, 2 is only in the audience on day 3, so 3 and 4 must perform on day 3. But
now 3 and 4 must be in the audience on day 1 in order to watch 1 and 2. That means they
cannot watch each other. So we need 4 days.

However, 4 days suffice for n = 6. For example:

1 2 3 watch 4 5 6
1 4 5 watch 2 3 6

2 4 6 watch 1 3 5

3 5 6 watch 1 2 4

Note that if m days suffice for n, then they also suffice for n-1 - just delete one group from the
schedule. So 4 days also suffice for 5. They must be necessary, because otherwise we could
do 4 groups in 3 days.
We can now do 12 groups in 6 days (and hence also 11 groups in 6 days):
1 2 3 A B C watch 4 5 6 D E F

1 4 5 A D E watch 2 3 6 B C F

2 4 6 B D F watch 1 3 5 A C E

3 5 6 C E F watch 1 2 4 A B D

1 2 3 4 5 6 watch A B C D E F

A B C D E F watch 1 2 3 4 5 6

Note that this is general. If m days suffice for n groups, then m+2 suffice for 2n groups. With
a slight variant we can do 10 groups in 5 days:
1 2 3 A B C watch 4 5 6 D

1 4 5 A B D watch 2 3 6 C

2 4 6 A C D watch 1 3 5 B

3 5 6 B C D watch 1 2 4 A

1 2 3 4 5 6 watch A B C D

So to finish off we have to show that 5 days are not sufficient for 11 groups. If we also show
that 4 are not sufficient for 7 groups, then we have dealt with all cases up to n = 12.
Consider n = 7. Suppose we only need 4 days. If a group only performs once, then the other 6
groups must watch that day. So none of them watch each other. So they need at least 4 days to
watch each other and only 3 are available. So each group must perform at least twice. But that
means on some day at least 4 groups perform (4 days x 3 groups < 7 groups x 2
performances). On that day none of the 4 can watch each other. But they need 4 days to watch
each other. So 7 groups need at least 5 days.
Consider n = 11. Suppose we only need 5 days. Each day at least 6 groups are watching or at
least 6 groups are performing. So there must be three days with at least 6 groups watching or
three days with at least 6 groups performing. But that means that there are 4 groups which all
perform on 2 days or which all watch on 2 days. In either case, they need a further 4 days to
watch each other. Contradiction. [The claim that if 6 groups watch on three days, then we can
find a particular set of 4 which watch on two of the three days is justified as follows. If only 3
watch on more than one day, then that leaves 8 who watch on just one day. So there is a day
when at most 2 of the 8 watch. But then on that day at most 2 + 3 can watch, whereas 6 are
supposed to.]
Summary:
n 3 4 5 6 7 8 9 10 11 12

days 3 4 4 4 5 5 5 5 6 6

14th CanMO 1982

Problem 1
Given a quadrilateral ABCD and a point P, take A' so that PA' is parallel to AB and of equal
length. Similarly take PB', PC', PD' equal and parallel to BC, CD, DA respectively. Show that
the area of A'B'C'D' is twice that of ABCD.

Solution
Note that direction is important. PA' is parallel to AB, not antiparallel.
We have that area PA'B' = area ABC, area PB'C' = area BCD, area PC'D' = area CDA and
area PD'A' = area DAB. In each case, the triangles are either congruent or have two sides the
same lengths but the angles between them summing to 180o (in which case their areas are the
same). But area ABC + area CDA = area BCD + area DAB = area ABCD, so area A'B'C'D' =
2 x area ABCD.

Problem 2

Show that the roots of x3 - x2 - x - 1 are all distinct. If the roots are a, b, c show that (a1982 -
b1982)/(a - b) + (b1982 - c1982)/(b - c) + (c1982 - a1982)/(c - a) is an integer.

Solution

Put f(x) = x3 - x2 - x - 1. We note that f(1) = -2, f(2) = 1, so there is a root between 1 and 2.
The other two roots are complex. However, that does not help much for a proof.

But assume the roots are not all distinct, so we have a repeated root a and the third root b
(which may also equal a). Then we have 2a + b = 1 (sum of roots), a2 + 2ab = -1 (product of
roots two at a time). Hence a2 + 2a(1 - 2a) + 1 = 0 or -3a2 + 2a + 1 = 0 or (3a + 1)(a - 1) = 0,
so a = 1 or -1/3. But it is easy to check that these are not roots. So the roots must all be
distinct.

Put h1 = (a + b + c) = 1, h2 = (ab + bc + ca) = -1, h3 = abc = 1. Put kn = (an - bn)/(a - b) + (bn -


cn)/(b - c) + (cn - an)/(c - a). We wish to show that all kn are integral.

We have k1 = 3, k2 = 2h1, k3 = 2h12 - 3h2, k4 = 2h13 - 5h1h2 + 3h3. We claim that kn+1 = h1kn -
h2kn-1 + h3kn-2. This is proved by straightforward algebra. For example, the numerators of the
terms with denominator (a - b) on the rhs are (a + b + c)(an - bn) - (ab + bc + ca)(an-1 - bn-1) +
abc(an-2 - bn-2) = an+1 - abn + anb - bn+1 + anc - bnc - anb + abn - an-1bc + bnc - anc + abn-1c + an-1bc -
abn-1c = an+1 - bn+1. Thus the desired conclusion (that all kn are integral follows by a trivial
induction).

Comment. Note that the recurrence relation is the same as that for the powers:

Put A = a + b + c, B = ab + bc + ca, C = abc, xn = an + bn + cn

x1 = A

x2 = A2 - 2B

x3 = A3 - 3AB + 3C

x4 = A4 - 4A2B + 4AC + 2B2

xn+1 = Axn - Bxn-1 + Cxn-2

Problem 3

What is the smallest number of points in n-dimensional space Rn such that every point of Rn is
an irrational distance from at least one of the points.

Solution

Answer: n = 1 number is 2. For n > 1, number is 3.

The result for n = 1 is obvious. The result for n > 1 follows from the fact that we can choose
three collinear points A, M, B such that the distance of any point P from one of them is
irrational. We can take a plane through P and A, M, B so the number of dimensions is
unimportant.

Let M be the midpoint of AB. Let PA = a, PB = b, PM = d, and AM = c. Let angle PMA = x.


Then the cosine rule gives a2 = d2 + c2 - 2dc cos x, b2 = d2 + c2 + 2dc cos x. So, adding, a2 + b2
= 2d2 + 2c2, so d2 = (a2 + b2)/2 - c2. Thus if we take c2 to be irrational, then if a and b are
rational, d2, and hence d, is irrational.

Problem 4

Show that the number of permutations of 1, 2, ... , n with no fixed points is one different from
the number with exactly one fixed point.

Solution

Put a(n) be the number of permutations with just 1 fixed point and d(n) the number with no
fixed points. We find that:
n 2 3 4

a(n) 0 3 8

d(n) 1 2 9

Evidently we wish to show that d(n) = a(n) + (-1)n (*). Note that it is almost obvious that a(n)
= n d(n-1). [If k is the fixed point, then the other n-1 points have no fixed points.] There are
two basic approaches. One is to establish the well-known formula d(n) = n! - n!/1! + n!/2! -
n!/3! + ... + (-1)nn!/n! . The other is to try to find a direct combinatorial argument for (*). The
latter is not promising, because it is well-known that the direct combinatorial argument for
d(n) = n d(n-1) + (-1)n is hard.
The formula is a simple application of the principle of inclusion and exclusion. Let cS(n) be
the number of permutations of 1, 2, ... , n which fix the points in the subset S. Evidently cS(n)
= (n - |S| )! (where nCk is the binomial coefficient), since we can permute the remaining n-|S|
points arbitrarily.
Hence the number of permutations with no fixed points is n! - nC1 (n-1)! + nC2 (n-2)! - ... +
(-1)n nCn 0! = n! (1 - 1/1! + 1/2! - 1/3! + ... + (-1)n/n! ). The required result now follows
immediately, for a(n) = n d(n-1) = n (n-1)! (1 - 1/1! + 1/2! - ... + (-1)n-1/(n-1)! ) = d(n) - (-1)n.
Thanks to Jacob Tsimerman for pointing out an error in the original proof.
The principle of inclusion and exclusion is as follows. Suppose we have N objects each of
which may have one or more of the properties a, b, c, ... . Let Na be the number with property
a, Nab the number with properties a and b, and so on. Then the number with none of these
properties is N - Na - Nb - ... + Nab + Nac + ... - Nabc - ... ± Nabc....

Problem 5

Let the altitudes of a tetrahedron ABCD be AA', BB', CC', DD' (so that A' lies in the plane
BCD and similarly for B', C', D'). Take points A", B", C", D" on the rays AA', BB', CC', DD'
respectively so that AA'·AA" = BB'·BB" = CC'·CC" = DD'·DD". Show that the centroid of
A"B"C"D" is the same as the centroid of ABCD.

Solution

Take vectors with origin at the centroid O of ABCD. Put A for the vector OA etc. The vector
(C - B) x (D - B) has direction along AA" and magnitude equal to the area of BCD or the
volume of ABCD divided by that length AA'. Hence the vector AA" is k (C - B) x (D - B) of
some k and the vector BB" is k (A - C) x (D - C) with the same k. Similarly, CC" is k (A - D)
x (B - D) and DD" is k (C - A) x (B - A). [Note that care is needed to get the order of the
factors correct in each case.] Thus we have to show that (C - B) x (D - B) + (A - C) x (D - C)
+ (A - D) x (B - D) + (C - A) x (B - A) = 0. Expanding, and remembering that X x X = 0 and
Y x X = -X x Y, we get C x D - C x B - B x D + A x D - A x C - C x D + A x B - A x D - D x
B + C x B - C x A - A x B = 0. Hence the centroid of A"B"C"D" is also O.
15th CanMO 1983

Problem 1

Find all solutions to n! = a! + b! + c! .

Solution

We must have n > a, b, c or n! < a! + b! + c! . But if n > a, b, c and n > 3, then n! > 3 (n-1)! ≥
a! + b! + c! . So we must have n = 1, 2, or 3. It is easy to check that this gives 3! = 2! + 2! + 2!
as the only solution.

Problem 2

Find all real-valued functions f on the reals whose graphs remain unchanged under all
transformations (x, y) → (2kx, 2k(kx + y) ), where k is real.

Solution

Answer: for any fixed real a, b we can take f(x) = x ln2|x| + ax for x positive and x ln2|x| + bx
for x negative.

Suppose f(0) = h. So (0, h) is a point on the graph. Hence so is (0, 2kh) for all k. But f is
single-valued, so h must be 0.

Suppose f(1) = h. Then (1, h) is a point on the graph. Hence so is (2k, 2kk + 2kh) for all real k.
Put k = ln2x. Then (x, x ln2x + xh) is a point on the graph. In other words, f(x) = x ln2x + hx.
We need to check that this works. So we have a point (x, x ln2x + hx). Under the
transformation this goes to the point P (2kx, 2k(kx + x ln2x + hx) ). Put x' = 2kx, so ln2x' = k +
ln2x. So we can write P as (x', x'(ln2x' - ln2x) + x' ln2x + hx') = (x', x' ln2x' + hx'), which is
indeed another point on the graph.

We now have to consider negative x. So suppose f(-1) = h. So (-1, h) is a point on the graph.
Hence so is (-2k, -2kk - 2kh) for all real k. Put k = ln2x. Then (-x, -x ln2x - hx) is a point on the
graph. In other words, f(x) = x ln2|x| + hx for x negative. Again we can easily check that this
works. Note that h does not have to be the same for positive and negative x.

Problem 3
Is the volume of a tetrahedron determined by the areas of its faces?

Solution

No. For example, take a tetrahedron with each side a 3, 4, 5 triangle (one pair of opposite
edges is 3, another pair is 4, and the third pair is 5). Then each face has area 6 and the volume
of the tetrahedron is nil. But if we take a regular tetrahedron with side 23/231/2 then each side
has area 6 but the volume is non-zero.

Problem 4

Show that we can find infinitely many positive integers n such that 2n - n is a multiple of any
given prime p.

Solution

We have 2p-1 = 1 mod p. So 2k(p-1) - k(p-1) = k+1 mod p. Thus if we take k to be hp - 1 for any
integer h, then we get 2k(p-1) - k(p-1) = 0 mod p. In other words n = (hp -1)(p - 1) works for any
h. For example, 24 - 4 = 12 = 0 mod 3, 210 - 10 = 1014 = 0 mod 3, 216 - 16 = 65520 = 0 mod 3
and so on.

Problem 5

Show that the geometric mean of a set S of positive numbers equals the geometric mean of the
geometric means of all non-empty subsets of S.

Solution

A geometric mean is a product xayb ... , where the exponents a, b, are positive reals with sum
1. This remains true if some of the x, y, ... are equal and we group like terms together, so that
not all the a, b, are equal. It also remains true if each of the x, y, ... is itself a geometric mean
and we express the product in terms of the underlying numbers. Because if (a + b + ... ) = 1, (c
+ d + ... ) = 1, ... and we take x(a + b + ... ) + y(c + d + ... ) + ... ,where x + y + ... = 1 and we
expand then we get a collection of terms with sum 1.

Thus if the elements of S are x, y, z, ... the geometric mean of the geometric mean of all non-
empty subsets of S simplifies to an expression of the form xayb ... where a + b + ... = 1. But by
symmetry we must have a = b = ... and hence the expression is just the geometric mean of S.

16th CanMO 1984


Problem 1
Show that the sum of 1984 consecutive positive integers cannot be a square.

Solution
(n+1) + (n+2) + ... + (n+1984) = (n+1984)(n+1985)/2 - n(n+1)/2 = 1984n + 1984.1985/2 =
992(2n + 1985). But 992 = 31.32 and (2n+1985) is odd, so the highest power of 2 dividing
992(2n+1985) is 32 = 25. Hence it is not a square
Problem 2

You have keyring with n identical keys. You wish to color code the keys so that you can
distinguish them. What is the smallest number of colors you need? [For example, you could
use three colors for eight keys: R R R R G B R R. Starting with the blue key and moving
away from the green key uniquely distinguishes each of the red keys.]

Solution

Answer: n = 2, n > 5 two colors. n = 3, 4 or 5, three colors.

For n even: take m red keys, then two blue keys, then n-m-3 red keys, then one blue key.
Provided m > n-m-3 this uniquely identifies each key. So this works for n > 5 (eg take m = 1).
For n = 1, only one color is needed. For n = 2, two colors are needed. For n = 3, we need three
colors, because if we only use two we cannot distinguish the keys with same color. Similarly
for n = 4, we need three colors (and three are sufficient eg RRGB - one red is next to blue and
the other is not). We also need three for n = 5, although that takes a little more effort to
demonstrate.

Problem 3

Show that there are infinitely many integers which have no zeros and which are divisible by
the sum of their digits.

Solution

We claim that the number an with 3n digits, all 1, is divisible by 3n. This is an easy induction.
It is true for n = 1: 111 = 3·37. But an+1 = an x 10...010...01 and 10...010...01 has sum of digits
3 and hence is divisible by 3, so an+1 is divisible by one more power of 3 than an.

Problem 4
An acute-angled triangle has unit area. Show that there is a point inside the triangle which is
at least 2/(33/4) from any vertex.

Solution

Let the triangle be ABC. Let R be the circumradius and O the circumcenter. Since the triangle
is acute-angled, O lies inside the triangle. We show that it is the desired point. Angle AOB =
2 angle C, so AB = 2R sin C. Similarly, BC = 2R sin A. Hence area ABC = (1/2) AB·BC sin
B = 2R2 sin A sin B sin C = 1. So it is sufficient to show that sin A sin B sin C ≤ 33/2/8. By the
AM/GM we have sin A sin B sin C ≤ (sin A + sin B + sin C)3/27. But sin x is convex for 0 <=
A, B, C ≤ 180o, so sin A + sin B + sin C ≤ 3 sin(A/3 + B/3 + C/3). One can also prove this
directly by noting that sin A + sin B = sin( (A+B)/2 + (A-B)/2 ) + sin( (A+B)/2 - (A-B)/2) = 2
sin(A+B)/2 cos(A-B)/2 ≤ 2 sin(A+B)/2 with equality iff A = B. Thus sin A sin B sin C ≤ sin3
60o = 33/2/8 as required.

Problem 5

Given any seven real numbers show we can select two, x and y, such that 0 ≤ (x - y)/(1 + xy)
≤ 1/√3.

Solution

Take the arctan of each number. The resulting 7 numbers lie in the open interval (-π/2, π/2).
Hence there must be two with difference < π/6. Suppose they are a >= b. Then 0 ≤ tan(a - b) <
1/√3. So if the corresponding original numbers are x = tan a, y = tan b, then x and y satisfy the
required inequalities.

17th CanMO 1985

Problem 1
A triangle has sides 6, 8, 10. Show that there is a unique line which bisects the area and the
perimeter.

Solution
Let such a line cut the sides length 10 and 8, distances x and y respectively from their
common vertex. Then the triangle with sides x and y has height 6x/10 and hence area 3xy/10.
That must equal 12, so xy = 40. Also x + y = 12. But then (x - y)2 = (x + y)2 - 4xy = -16, so
there are no such solutions.
If the line cuts the sides length 6 and 8, distances x and y from their common vertex, then we
have xy = 24, x + y = 12. Hence x, y = 6 ± 2radic;3. But 6 + 2√3 > 6 and 8, so there are no
such solutions.
The final possibility is that the line cuts the sides length 10 and 6, distances x and y
respectively from their common vertex. Then xy = 30, x + y = 12, so x, y = 6 ± √6. The larger
value exceeds 6, so we must have x = 6 + √6, y = 6 - √6, and the line is unique.

Problem 2

Is there an integer which is doubled by moving its first digit to the end? [For example, 241
does not work because 412 is not 2 x 241.]

Solution

Suppose such a number has first digit a, and n other digits. Then it is 10na + N for some N
with n digits. Moving the first digit to the end transforms it to 10N + a, so we require 10N + a
= 2(10na + N). Hence 8N = (2.10n - 1)a. Now 2.10n - 1 is odd, so 8 must divide a. Hence a = 8.
So N = 2.10n - 1. But 2.10n - 1 > 10n and hence has n+1 digits. Contradiction. So there is no
such number.

Problem 3

A regular 1985-gon is inscribed in a circle (so each vertex lies on the circle). Another regular
1985-gon is circumcribed about the same circle (so that each side touches the circle). Show
that the sum of the perimeters of the two polygons is at least twice the circumference of the
circle. [Assume tan x ≥ x for 0 ≤ x < 90o.]

Solution

Let the circle have radius R and let a side of the inner polygon subtend an angle 2x at the
center. Then the side has length 2R sin x. The side of the outer polygon is 2R tan x. So it is
sufficient to show that sin x + tan x > 2x.

We have sin x = 2t/(1 + t2), tan x = 2t/(1 - t2), where t = tan(x/2). Hence sin x + tan x = 4t/(1 -
t4) > 4t > 4 x/2 = 2x.

Problem 4

Show that n! is divisible by 2n-1 iff n is a power of 2.


Solution

We claim that the highest power of 2 dividing n! is 2n-b(n), where b(n) is the number of 1s in
the binary expression for 2. This can easily be proved by induction. For n = 1 it is obvious. So
suppose it is true for n. If n+1 is odd, then the highest power of 2 dividing (n+1)! is the same
as that dividing n! and n+1 has the same binary digits as n except that the final 0 is replaced
by 1, so n+1 - b(n+1) = n - b(n) and the result is true for n+1. Suppose n+1 is even and that 2m
is the highest power of 2 dividing n+1. Then the highest power dividing (n+1)! is m higher
than that dividing n! . But n must have binary expansion ...011...1 (with m final 1s) and n+1
the expansion ...10...0 (with m final 0s). So b(n+1) = b(n) - m + 1. Hence n+1 - b(n+1) = n -
b(n) + m. In other words, n+1 - b(n+1) is m larger than n - b(n). So the result is true for n+1.
Hence the result is true for all n.

Finally, note that b(n) = 1 iff n is a power of 2.

Problem 5

Define the real sequence x1, x2, x3, ... by x1 = k, where 1 < k < 2, and xn+1 = xn - xn2/2 + 1.
Show that |xn - √2| < 1/2n for n > 2.

Solution

f(x) = x - x2/2 + 1 = 3/2 - (x - 1)2/2, which is a strictly decreasing function over the range x = 1
to x = 2. Also, f(1) = 1.5, f(2) = 1 and f(1.5) = 11/8 = 1.375. Thus 1 < x2 < 1.5 and 1.375 < x3
< 1.5. We have √2 - 1/8 = 1.29, √2 + 1/8 = 1.53. So the required result is true for n = 3. We
show that it is true for larger n by induction. Suppose it is true for n. Then xn+1 < f(√2 - 1/2n) =
√2 - 1/2n - (√2 - 1/2n)2/2 + 1 = √2 - 1/2n + (√2)/2n - 1/22n+1 < √2 + (√2 - 1)/2n < √2 + 1/2n+1
(since √2 < 1.5). Similarly, xn+1 > f(√2 + 1/2n) = √2 + 1/2n - (√2)/2n - 1/22n+1 = √2 - k/2n, where
k = √2 + 1/2n+1 - 1. Now n+1 >= 4 and √2 + 1/16 < 1.5, so k < 1/2. Hence xn+1 > √2 - 1/2n+1,
which completes the induction.

18th CanMO 1986

Problem 1
The triangle ABC has ∠B = 90o. The point D is taken on the ray AC, the other side of C from
A, such that CD = AB. The ∠CBD = 30o. Find AC/CD.
Solution
Let angle ACB = x. Wlog we may take AC = 1. Then BC = cos x and AB = sin x. So CD =
sin x. Now apply the sine rule to the triangle BCD: cos x/(sin(x-30o) = sin x/sin 30o. Hence
cos x = √3 sin2x - sin x cos x. Hence (1 + sin x)2(1 - sin2x) = 3 sin4x. So 4 sin4x + 2 sin3x - 2
sin x - 1 = 0. Factorising, (2 sin3x - 1)(2 sin x + 1) = 0. But x must lie between 0 and 90o, so
we cannot have sin x = -1/2. Hence sin x = 1/21/3. Hence the ratio AC/CD = 21/3.

Problem 2

Three competitors A, B, C compete in a number of sporting events. In each event a points is


awarded for a win, b points for second place and c points for third place. There are no ties.
The final score was A 22, B 9, C 9. B won the 100 meters. How many events were there and
who came second in the high jump?

Solution

A 5, 5, 5, 5, 2; B 5, 1, 1, 1, 1; C 2, 2, 2, 2, 1. A must have come second in the event in which


B won. So C came second in every other event.

Problem 3

A chord AB of constant length slides around the curved part of a semicircle. M is the
midpoint of AB, and C is the foot of the perpendicular from A onto the diameter. Show that
angle ACM does not change.

Solution

Let O be the center of the semicircle. Angle OMA = angle OCA = 90o, so OMAC is cyclic.
Hence angle ACM = angle AOM, which is independent of the position of the chord since it
has constant length.

Problem 4

Show that (1 + 2 + ... + n) divides (1k + 2k + ... + nk) for k odd.

Solution

Assume k is odd and let N = (1k + 2k + ... + nk). We have 2N = (nk + 1k) + ( (n-1)k + 2k) + ... +
(1k + nk). But expanding (n+1 - m)k by the binomial theorem we find that it is (-m)k mod n+1
and hence mk + (n+1 - m)k is divisible by n+1. So n+1 divides 2N. Similarly, we may write
2N = 2nk + ( (n-1)k + 1k) + ( (n-2)k + 2k) + ... + (1k + (n-1)k) and each term is divisible by n.
But n and n+1 are relatively prime, hence n(n+1) divides 2N. Hence 1 + 2 + ... + n = n(n+1)/2
divides N.

Problem 5

The integer sequence a1, a2, a3, ... is defined by a1 = 39, a2 = 45, an+2 = an+12 - an. Show that
infinitely many terms of the sequence are divisible by 1986.

Solution

We find a3 = 1986. Define bn = residue of an mod 1986. Then bn satisfies the relation bn+2 =
bn+12 - bn mod 1986. Now there are only 19862 possibile values for the pair (bn, bn+1), so some
pair must repeat. But any pair of consecutive values of bn determines the rest of the sequence
and all preceding terms of the sequence (since bn = bn+12 - bn+2). So the values of bn must be
periodic for some period N <= 19862. We know that the value 0 occurs for n = 3, so it must
occur infinitely often.

Comment. But it takes a while, the values 39, 45 next occur at b1339 , b1340 .

19th CanMO 1987

Problem 1
Find all positive integer solutions to n! = a2 + b2 for n < 14.

Solution
Answer: 2! = 12 + 12, 6! = 242 + 122.
A necessary condition for N to be the sum of two squares is that N does not have a prime p =
3 mod 4 such that the highest power of p dividing N is odd. Thus for n = 3, 4, 5 the highest
power of 3 dividing n! is 31. For n = 7, 8, 9, 10, 11, 12, 13 the highest power of 7 dividing n!
is 71. So in all those cases n! cannot be written as a sum of two squares. Obviously 1! cannot
(because the smallest sum of two squares is 2) and we have exhibited 2! and 6! as a sum of
two squares.
To prove the criterion, note that if p divides N and N = a2 + b2 then a2 = -b2 mod p, so if a and
b are not multiples of p, then we can take c such that ac = 1 and get x2 = -1 mod p, where x =
bc. But if p = 3 mod 4, then (p-1)/2 is odd, so xp-1 = (-1)odd = -1 mod p, whereas we know that
xp-1 = 1 mod p for all x not zero mod p.

Problem 2

Find all the ways in which the number 1987 can be written in another base as a three digit
number with the digits having the same sum 25.

Solution

452 = 2025. So any three digit number in base 45 or higher is too big. 123 = 1728, so any three
digit number in base 12 or less is too small. Suppose the base is b and the number has first
two digits m and n. Then we have mb2 + nb + (25 - m - n) = 1987. So m(b2 - 1) + n(b - 1) =
1962 = 18·109. Hence b - 1 divides 18.109. But 109 is prime, so we must have b = 19. We
have 1987 = 5·192 + 9·19 + 11 and 5 + 9 + 11 = 25, so b = 19 works.

Problem 3

ABCD is a parallelogram. X is a point on the side BC such that ACD, ACX and ABX are all
isosceles. Find the angles of the parallelogram.

Solution

There are 5 possible configurations. (1) ∠D = 36o, CA = CD = CX, XB = XA, (2) ∠D = 36o,
DA = DC, AC = AX = BX, (3) ∠D = 36o, CA = CD, XA = XC, BA = BX, (4) ∠D = 45o, CD
= CA, XC = XA = XB, (5) ∠D = 77 1/7o, AC = AD, XC = XA, BA = BX.

One just has to go through the various possibilities. In triangle ABX the equal angles could be
at A and B, B and X, or X and A. Case 1, ∠XAB = ∠XBA = x. Hence angle D = x, ∠CXA =
2x, ∠XCD = 180o - x. Now in triangle AXC, the equal angles could be at X and A or various
other possibilities. Suppose they are at X and A. Then we get all the angles in terms of x. Now
require that ACD is isosceles to get x. Similarly for the other cases.

Problem 4
n stationary people each fire a water pistol at the nearest person. They are arranged so that the
nearest person is always unique. If n is odd, show that at least one person is not hit. Does one
person always escape if n is even?

Solution

n odd. Use induction. For n = 3, let the people be A, B, C. Let AB be the shortest side of the
triangle. Then A and B shoot each other and so no one shoots C. Suppose it is true for n.
Given n+2 people let A, B be the closest pair (or one of them). Then A and B shoot each
other. If anyone else shoots A or B, then the victim is hit twice. There are only n+2 shots, so if
one person gets hit more than once, then there are at most n shots left between n+1 people, so
someone must escape. If no one else shoots at A and B, then by induction one of the
remaining n people escapes.

For n even, take isolated pairs of people, then each pair wet each other and no one escapes.
On the other hand someone obviously can escape. For example, take everyone close together
except for one person - no one shoots at him.

Problem 5

Show that [√(4n + 1)] = [√(4n + 2)] = [√(4n + 3)] = [√n + √(n + 1)] for all positive integers n.

Solution

Put f(n) = [√n + √(n + 1)]. Obviously f(n) is monotonic increasing. We have f(m2 - 1) = 2m - 1
and f(m2) = 2m. Suppose n = m(m+1)-1. Then (√n + √(n + 1))2 = 2m(m+1)-1 + 2√( (m(m+1)-
1) m(m+1) ) < 2m(m+1) - 1 + 2m(m+1) < (2m+1)2. So f(n) = 2m. But if n = m(m+1), then (√n
+ √(n + 1))2 > 2m(m+1)+1 + 2m(m+1) = (2m+1)2, so f(n) = 2m+1. Thus f(n) increments by 1
at n = m2 and m2 + m.

Now for n = m2 - 1, 4n+1 = 4m2 - 3, ... , 4n+3 = 4m2 - 1, so [√(4n + 1)] = [√(4n + 2)] = [√(4n
+ 3)] = 2m-1. But for n = m2, 4n+1 = 4m2+1, 4n+2 = 4m2+2, 4n+3 = 4m2+3, so [√(4n + 1)] =
[√(4n + 2)] = [√(4n + 3)] = 2m. Similarly for n = m(m+1)-1, 4n+1 = 4m2+4m-3 = (2m+1)2-4,
4n+2 = (2m+1)2-3, 4n+3 = (2m+1)2-2, so [√(4n + 1)] = [√(4n + 2)] = [√(4n + 3)] = 2m. But
for n = m(m+1), 4n+1 = (2m+1)2 etc and so [√(4n + 1)] = [√(4n + 2)] = [√(4n + 3)] = 2m+1.

Obviously [√(4n + 1)], [√(4n + 2)], [√(4n + 3)] are all monotonic increasing, so they all equal
f(n).

20th CanMO 1988


Problem 1
For what real values of k do 1988x2 + kx + 8891 and 8891x2 + kx + 1988 have a common
zero?

Solution
If x is a common zero, then 1988x2 + kx + 8891 = 0, and 8891x2 + kx + 1988 = 0. Subtracting
one equation from the other gives 6903(x2 - 1) = 0, so x = 1 or -1. Substituting back into the
first equation gives k = ±10879. Check: for k = 10879, the polynomials are (1998x + 8891)(x
+ 1) and (8891x + 1998)(x + 1); or k = -10879, the polynomials are (1998x - 8891)(x - 1) and
(8891x - 1998)(x - 1). In both cases, there is indeed a common zero.

Problem 2

Given a triangle area A and perimeter p, let S be the set of all points a distance 5 or less from
a point of the triangle. Find the area of S.

Solution

S is the triangle itself (and all its interior points), a rectangle width 5 on each side and the
sector of a circle radius 5 at each vertex to join together the two rectangles. The three sectors
together form a circle. Hence the area of S is A + 5p + 25 π.

Problem 3

Given n > 4 points in the plane, some of which are colored red and the rest black. No three
points of the same color are collinear. Show that we can find a three points of the same color,
such that two of the points do not have a point of the opposite color on the segment joining
them.

Solution

There are at least 5 points, so there must be at least 3 of the same color. Label them A1, B1, C1.
If one pair does not have a point of the opposite color on the segment joining them, then we
are done. If not, take the three points of the opposite color to be A2, B2, C2. Now repeat. The
process must terminate because at each step we get three new points (none of the previous
points can lie on the sides of AnBnCn, because all points of AnBnCn except the vertices lie
inside the triangle An-1Bn-1Cn-1). So for some n An, Bn, Cn are the required points.

Problem 4
Define two integer sequences a0, a1, a2, ... and b0, b1, b2, ... as follows. a0 = 0, a1 = 1, an+2 = 4an+1
- an, b0 = 1, b1 = 2, bn+2 = 4bn+1 - bn. Show that bn2 = 3an2 + 1.

Solution

We note that a2 = 4 and b2 = 7. Let Sn be the statement that bn2 = 3an2 + 1 and bnbn-1 = 3anan-1 +
2. So S1 is true, because b12 = 4 = 3a12 + 1 and b1b0 = 2 = 3a1a0 + 2. S2 is also true, because b22
= 49 = 3a22 + 1 and b2b1 = 14 = 3a2a1 + 2.

We show that Sn is true for all n by induction. Suppose it is true for n and n-1. We have bn+1bn
= bn(4bn - bn-1) = 4bn2 - bnbn-1 = 12an2 + 4 - (3anan-1 + 2) = 3an(4an - an-1) + 2 = 3anan+1 + 2, as
required. Similarly, bn+12 - 3an+12 = (4bn - bn-1)2 - 3(4an - an-1)2 = 16(bn2 - 3n2 - 1) - 8(bnbn-1 - 3anan-
2 2
1 - 2) + bn-1 - 3an-1 = 0 - 0 + 1, as required.

Problem 5

If S is a sequence of positive integers let p(S) be the product of the members of S. Let m(S) be
the arithmetic mean of p(T) for all non-empty subsets T of S. S' is formed from S by
appending an additional positive integer. If m(S) = 13 and m(S') = 49, find S'.

Solution

Suppose there are n elements in S, namely a1, a2, ... , an. Then m(S) = (K - 1)/(2n - 1), where K
= (a1 + 1)(a2 + 1) ... (an + 1). If the additional integer is h, then m(S') = (K(h + 1) - 1)/(2n+1 - 1).
So we have K = 13(2n - 1) + 1, (h + 1)K = 49(2n+1 - 1) + 1. Hence h + 1 = (49·2n+1 - 48)/(13·2n
- 12). But 7(13·2n - 1) = 91·2n - 84 < 98.2n - 48 = 49·2n+1 - 48. For n > 3, 8(13·2n - 12) =
(52·2n+1 - 96) > (49·2n+1 - 48). So we must have n = 1, 2, or 3. If n = 1, then h+1 = 148/14,
which is not an integer. If n = 2, then h+1 = 344/40, which is not an integer. If n = 3, then h+1
= 736/92 = 8, which is ok.

If n = 3, then if the elements of S are a, b, c, we must have (a + 1)(b + 1)(c + 1) = 92 = 2.2.23.


So S = {2, 2, 23} and S' = {2, 2, 7, 23}.

21st CanMO 1989

Problem 1
How many permutations of 1, 2, 3, ... , n have each number larger than all the preceding
numbers or smaller than all the preceding numbers?
Solution
Answer: 2n-1.
Let the number of permutations be a(n). The last number must be 1 or n, for if it is k with 1 <
k < n, then it is preceded by 1 < k and by n > k. If the last number is n, then the preceding
numbers satisfy the conditions for a permutation of 1, 2, ... , n-1. So there are a(n-1)
permutations with last number n. Similarly, there are a(n-1) with last number 1. Hence a(n) =
2a(n-1). But a(2) = 2, so a(n) = 2n-1.

Problem 2

Each vertex of a right angle triangle of area 1 is reflected in the opposite side. What is the area
of the triangle formed by the three reflected points?

Solution

Answer: 3.

Let the original triangle be ABC with angle B = 90o. Let the reflection of A be A' and so on.
Then ABA' and CBC' are straight lines and B is the midpoint of AA' and CC'. So C'A' is
parallel to AC and of equal length. Also BB' is perpendicular to AC and to C'A'. Hence the
altitude from B' to C'A' is three times the length of the altitude from B to AC. So the area of
A'B'C' is three times that of ABC.

Problem 3

Tranform a number by taking the sum of its digits. Start with 19891989 and make four
transformations. What is the result?

Solution

Let f(n) be the sum of the digits of n. We have 19891989 < 104·1989, so f(n) < 4·1989·9 = 71604
< 99999, so f(f(n)) < 45. Hence f(f(f(n))) <= 12 and f(f(f(f(n)))) <= 9. But f(n) = n mod 9.
Now 1989 = 0 mod 9, so 19891989 = 0 mod 9. Hence f(f(f(f(n)))) = 0 mod 9. Obviously f(n) >
0 if n > 0, so the result of the four tranformations must be 9.

Problem 4

There are five ladders. There are also some ropes. Each rope attaches a rung of one ladder to a
rung of another ladder. No ladder has two ropes attached to the same rung. A monkey starts at
the bottom of each ladder and climbs. Each time it reaches a rope, it traverses the rope to the
other ladder and continues climbing up the other ladder. Show that each monkey eventually
reaches the top of a different ladder.

Solution

We have to show two separate things: that each monkey reaches the top of some ladder and
that no two monkeys reach the top of the same ladder. Let us start with the second.

Let a section be the part of a ladder between two consecutive ropes. We claim that only one
monkey can ever visit a given section. Suppose that was false. So suppose monkeys A and B
both visit a particular section. Let S be the first section which A visits which is also visited by
B. This cannot be the initial section of A's starting ladder, because B cannot get to that and it
cannot be the initial section of any other ladder, because then A could not get to it. So it must
be a section which has a rope R at its lowest point. A and B must both come along R to reach
S, because if they came up the ladder from below they would have to go along R rather than
S. But that means they must both have visited the section ending at the other end of the rope,
contradicting the definition of S. That establishes the claim. In particular, only one monkey
can visit the final section on a ladder, above the top rope, so at most one monkey can reach
the top of any given ladder.

Similarly, we claim that a monkey can visit any given section at most once. Suppose
otherwise. Let S be the first section that A visits that he later revisits. Let R be the rope at the
start of S (as above, there must be such a rope). So on both visits A must have come along R,
but that means he visited the section ending at the other end of the rope twice, contradicting
the definition of S. That establishes the claim. But there are only a finite number of sections,
so each monkey must eventually stop and that means it must reach the top of a ladder.

Problem 5

For every permutation a1, a2, ... , an of 1, 2, 4, 8, ... , 2n-1 form the product of all n partial sums
a1 + a2 + ... + ak. Find the sum of the inverses of all these products.

Solution

Answer: 1/2N, where N = (n-1)n/2.

For example, we have:

n = 2: 12 partial sums 13 1/3

21 partial sums 23 1/6

total 1/2
n = 3: 124 partial sums 137 1/21

142 partial sums 157 1/35

214 partial sums 237 1/42

241 partial sums 267 1/84

412 partial sums 457 1/140

421 partial sums 467 1/168

total 1/8

We claim the more general result that if each ai is a power of 2 with exponent bi, then the sum
of the inverses is 2-B, where B is the sum of the bi. We use induction on n. This is obvious for
n = 1. Suppose it is true for < n. Let A be the sum of the ak. The sum of the inverses over the
permutations which have ak last is (sum for the n-1 numbers excluding ak) x 1/A = ( 1/2 to the
power of (B - bk) )/A. Hence the sum of all the inverses is (1/(A 2B) x sum of 2 to the power of
bk = (1/(A 2B) x sum of ak = 1/2B, which establishes the induction.

22nd CanMO 1990

Problem 1
A competition is played amongst n > 1 players over d days. Each day one player gets a score
of 1, another a score of 2, and so on up to n. At the end of the competition each player has a
total score of 26. Find all possible values for (n, d).

Answer
(n, d) = (3, 13), (12, 4) or (25, 2)

Solution
Each day n(n+1)/2 points are scored, so dn(n+1)/2 over the whole competition. This equals
26n, so d(n+1) = 52. So d = 1, 2, 4, 13 or 52. But d = 1 does not work, because one player
scores 1 not 26. d = 2 (n = 25) does work: one player scores 1, 25 on successive days, the next
2, 24, the third 3, 23 and so on. d = 4 (n = 12) also works. For example, the players' scores
could be:
day 1 1 2 3 4 5 6 7 8 9 10 11 12

day 2 12 11 10 9 8 7 6 5 4 3 2 1

day 3 1 2 3 4 5 6 7 8 9 10 11 12

day 4 12 11 10 9 8 7 6 5 4 3 2 1

d = 13 (n = 3) also works (in the first three days each players scores 6, then in each 5
successive pairs of days he scores 4):
day 1 2 3 4 5 6 7 8 9 10 11 12 13
player 1 1 2 3 1 3 1 3 1 3 1 3 1 3

player 2 2 3 1 3 1 3 1 3 1 3 1 3 1

player 3 3 1 2 2 2 2 2 2 2 2 2 2 2

d = 52 does not work, because there are no players!

Problem 2

n(n + 1)/2 distinct numbers are arranged at random into n rows. The first row has 1 number,
the second has 2 numbers, the third has 3 numbers and so on. Find the probability that the
largest number in each row is smaller than the largest number in each row with more
numbers.

Solution

Answer: 2n/(n+1)! .

Let the probability be pn. The largest number must go into the last row. The probability of that
happening is n/( n(n+1)/2 ) = 2/(n+1). It is then of no consequence what other numbers go in
the last row. But the (n-1)n/2 numbers that go in the other n-1 rows must meet the criterion for
n-1. The probability of that happening is pn-1. Hence pn = 2 pn-1/(n+1). Also p1 = 1. So by a
trivial induction pn = 2n/(n+1)! .

Problem 3

The feet of the perpendiculars from the intersection point of the diagonals of a convex cyclic
quadrilateral to the sides form a quadrilateral q. Show that the sum of the lengths of each pair
of opposite sides of q is equal.

Solution
Let the quadrilateral be ABCD. Let the diagonals meet at X. Let the perpendiculars from X to
AB, BC, CD, DA have feet at P, Q, R, S. Then ∠XPA = ∠XSA = 90o. So XPSA is cyclic.
ABCD is also cyclic and ∠PAS = ∠BAD, so PS/(diam XPSA) = BD/(diam ABCD). But the
diameter of XPSA is AX. Let the diameter of ABCD be 2R, then PS/BD = AX/2R. Similarly,
QR/BD = CX/2R, so (PS + QR)/BD = (AX + XC)/2R, or PS + QR = AC·BD/2R. Similarly,
PQ + RS = BD·AC/2R. Hence result.

Problem 4

A particle can travel at a speed of 2 meters/sec along the x-axis and 1 meter/sec elsewhere.
Starting at the origin, which regions of the plane can the particle reach within 1 second.

Solution

Let O be the origin, B the point (2, 0), A the point (1/2, (√3)/2, and C the point (1, 0). We
show that the region which can be reached in the first quadrant is any point in the triangle
OAB and any point in the sector with center O and arc AC. The regions which can be reached
in the other quadrants are obtained by reflecting this region in the two axes.

We consider first the best way of reaching the point P (a, b) in the first quadrant. Evidently it
is to travel a distance (a - x) along the x-axis and then in a straight line to (a, b). So the time
required is (a - x)/2 + (b2 + x2)1/2. We claim that this expression is at least a/2 + (b√3)/2 with
equality iff x = b/√3. This is equivalent to showing that b2 + x2 ≥ (x/2 + (b√3)/2)2 or 3x2 -
2xb√3 + b2 ≥ 0, or (x√3 - b)2 >= 0, which is evidently true. Thus if a > b/√3, then one travels
along the x-axis to X, where the angle BXP = 60o and then directly to P. If angle POB > 60o,
then one goes directly from O to P.

If one goes to X and then P, with angle BXP = 60o, then P must lie inside the triangle ABC.
For take the point Q at the intersection of AB and XP. Triangles BXQ and BAO are similar,
so XQ = AO·BX/OB =1·BX/2. Hence OX/2 + XQ = OB/2 = 1. In other words it takes exactly
time 1 to reach Q. Hence P must lie on XQ.

Problem 5

N is the positive integers, R is the reals. The function f : N → R satisfies f(1) = 1, f(2) = 2 and
f(n+2) = f(n+2 - f(n+1) ) + f(n+1 - f(n) ). Show that 0 ≤ f(n+1) - f(n) <= 1. Find all n for
which f(n) = 1025.
Solution

Calculating the first few values we find f(3) = 2, f(4) = 3, f(5) = f(6) = f(7) = 4, f(8) = 5, f(9) =
f(10) = 6, f(11) = 7, f(12) = f(13) = f(14) = f(15) = 8, f(16) = 9. That suggests the pattern f(2n)
= 2n-1 + 1 and that the value m occurs k times, where 2k-1 is the highest power of 2 dividing m.

23rd CanMO 1991

Problem 1
Show that there are infinitely many solutions in positive integers to a2 + b5 = c3.

Solution
We have 102 + 35 = 73. Hence (10n15)2 + 3(n6)5 = (7n10)3 for all n.

Problem 2

Find the sum of all positive integers which have n 1s and n 0s when written in base 2.

Solution

The first digit must be 1. Then the remaining 2n-1 digits are subject only to the constraint that
n are 0 and n-1 are 1. So there are (2n-1)!/(n! (n-1)! ) possibilities. There are (2n-2)!/( ( n! (n-
2)! ) with a 1 in any given position (apart from the first). Thus the sum is (2n-1)!/(n! (n-1)! )
22n-1 + (2n-2)!/( n! (n-2)! ) (1 + 2 + 22 + ... + 22n-2) = (2n-1)!/(n! (n-1)! ) 22n-1 + (2n-2)!/( n! (n-
2)! ) (22n-1 - 1). For n = 1 the formula breaks down, but in that case the sum is obviously 2.

Problem 3

Show that the midpoints of all chords of a circle which pass through a fixed point lie on
another circle.

Solution

Let the circle have center O and let the point be X. We have angle OMX = 90o, so M lies on
the circle diameter OX. [We are not asked to find the locus of M, simply to show that the
locus is part of a circle. In fact, if X lies outside the circle, then the locus is the part of the
circle diameter OX which lies inside the given circle. If X lies inside, then it is the complete
circle diameter OX.]

Problem 4

Can ten distinct numbers a1, a2, b1, b2, b3, c1, c2, d1, d2, d3 be chosen from {0, 1, 2, ... , 14}, so
that the 14 differences |a1 - b1|, |a1 - b2|, |a1 - b3|, |a2 - b1|, |a2 - b2|, |a2 - b3|, |c1 - d1|, |c1 - d2|, |c1 -
d3|, |c2 - d1|, |c2 - d2|, |c2 - d3|, |a1 - c1|, |a2 - c2| are all distinct?

Solution

Answer: no.

We use a parity argument. There are 14 possible non-zero differences between numbers from
the set {0, 1, 2, ... , 14}, namely 1, 2, ... , 14. Since the 14 differences are required to be all
different, they must be 1, 2, ... , 14 (in some order). So exactly 7 of the differences must be
odd. We show that this is impossible.

Suppose a1 and a2 have the same parity, and c1 and c2 have the same parity. Then either 0 or 2
of |a1 - c1|, |a2 - c2| are odd. But for each bi, either both |a1 - bi|, |a2 - bi| are odd or neither are
odd. So an even number of differences |ai - bj| are odd. Similarly, an even number of
differences |ci - dj| are odd. So we must have an even number of odd differences.
Contradiction.

Suppose a1 and a2 have opposite parity, and c1 and c2 have opposite parity. Again, an even
number of |ai - ci| are odd. But now just one of |a1 - bi|, |a2 - bi| is odd, so three of |ai - bj| are
odd, and similarly three of |ci - dj|, so an even number of odd differences in total.
Contradiction.

Suppose a1 and a2 have the same parity, but c1 and c2 have opposite parity. Then just one of |ai
- ci| is odd. But an even number of |ai - bj| are odd, whilst three of |ci - dj| are odd. So again we
have an even number of odd differences in all. Contradiction. Similarly, if a1 and a2 have
opposite parity, whilst c1 and c2 have the same parity.

Problem 5

An equilateral triangle side n is divided into n2 equilateral triangles side 1 by lines parallel to
its sides. How many parallelograms can be formed from the small triangles? [For example, if
n = 3, there are 15, nine composed of two small triangles and six of four.]

Solution
Answer: (n+2)(n+1)n(n-1)/8 = 3 (n+2)C4.

The parallelogram must have angles 60o, 120o, 60o, 120o. The 60o angles can be aligned in
three ways (with one of the vertices of the triangle) and the number of parallelograms for each
way is obviously equal. Suppose it is an (so that the total number of parallelograms is 3 an).

Let the big triangle be ABC. Let bn be the number of parallelograms where one vertex of the
parallelogram is A. Then we evidently have an = bn + 2 bn-1 + 3 bn-2 + ... + (n-1) b2. But it is
easy to calculate bn directly. Such a parallelogram is uniquely defined by the position of the
vertex opposite to A. But that can have any position in the rows of vertices (of the small
triangles) provided it is at least two rows below A and not at either end. Thus there are 1 + 2 +
... + n-1 = n(n-1)/2 possible positions for it. So bn = n(n-1)/2 and bn+1 - bn = n. Hence an+1 - an =
1.n + 2(n-1) + 3(n-2) + ... + n.1 = n(1 + 2 + ... + n) - (1.2 + 2.3 + ... + (n-1)n) = n2(n+1)/2 - (n-
1)n(n+1)/3 = n(n+1)(n+2)/6 = (n+2)C3. Now we claim that an = (n+2)C4. It is true for n = 2
(obviously a2 = 1). So the result is a trivial induction, since (n+2)C4 + (n+2)C3 = (n+3)C4.
Hence, finally, the total number of parallelograms is 3an = 3 (n+2)C4.

24th CanMO 1992

Problem 1
Show that n! is divisible by (1 + 2 + ... + n) iff n+1 is not an odd prime.

Solution
For n = 1, we have to show that 1 divides 1, which is obvious. So assume n > 1. We have 1 +
2 + ... + n = n(n+1)/2. So we have to show that n+1 divides (n-1)! for n+1 not a prime. But
that is obvious except possibly in the case n+1 = p2. However, in that case p and 2p < n, so p2
still divides (n-1)! . [The converse, that if n+1 is an odd prime then it does not divide n!, is
obvious.]

Problem 2

Show that x(x - z)2 + y(y - z)2 >= (x - z)(y - z)(x + y - z) for all non-negative reals x, y, z.
When does equality hold?

Solution

Answer: equality iff x = y = z or x = y and z = 0.

Put u = x - z, v = y - z. Then x(x - z)2 + y(y - z)2 - (x - z)(y - z)(x + y - z) = (u + z)u2 + (v +


z)v2 - uv(u + v + z) = (u + v)(u - v)2 + z(u2 - uv + v2). If u and v are both non-negative, then
this is obviously non-negative (since u2 - uv + v2 = (u - v)2 + uv) and zero only if u = v = 0 or
u = v and z = 0 (in other words, x = y = z or x = y and z = 0).
If just one of u, v is negative, then the rhs of the original expression is negative or zero and
the rhs is positive, so strict inequality holds.

If both u and v are negative, then we may assume x + y > z (otherwise the lhs of the original
expression is positive and the rhs negative or zero). But in that case z > -(u + v) and u2 - uv +
v2 > (u - v)2, so z(u2 - uv + v2) > -(u + v)(u - v)2.

Problem 3

ABCD is a square. X is a point on the side AB, and Y is a point on the side CD. AY meets
DX at R, and BY meets CX at S. How should X and Y be chosen to maximise the area of the
quadrilateral XRYS?

Solution

Answer: the maximum area of (area ABCD)/4 is achieved whenever XY is parallel to AD.

Area XRYS = area XRS + area YRS. Area XRS = (XR/XD) area XSD = (XR/XD) (XS/XC)
area XDC = (XR/XD) (XS/XC) (area ABCD)/2. Similarly, area YRS = (YS/BY) (RY/AY)
(area ABCD)/2. But XR/XD = 1 - YR/AY, XS/XC = 1 - YS/BY. So if we put XR/XD = (1/2
+ x), XS/XC = (1/2 + y), then area XRYS = (1/4 + xy) area ABCD.

But triangles ARX and YRD are similar, so XR > DR iff AX > YD. Similarly, XS > SC iff
BX > CY. Hence XR/XD > 1/2 iff XS/XC < 1/2 and XR/XD = 1/2 iff XS/XC = 1/2. Thus
either x = y = 0 or xy is negative. If x = y = 0, then XY is parallel to AD.

Problem 4

Find all real solutions to x2(x + 1)2 + x2 = 3(x + 1)2.

Solution

Expanding, x4 + 2x3 - x2 - 6x - 3 = 0. Factorising, (x2 - x - 1)(x2 + 3x + 3) = 0. The first


quadratic has real roots x = 1/2 ± (√5)/2. The second quadratic is (x + 3/2)2 + 3/4, so has no
real roots.

Problem 5

There are 2n+1 cards. There are two cards with each integer from 1 to n and a joker. The
cards are arranged in a line with the joker in the center position (with n cards each side of it).
For which n < 11 can we arrange the cards so that the two cards with the number k have just
k-1 cards between them (for k = 1, 2, ... , n)?

Solution

Consider the number of pairs (i, j) where i lies between the two j's or j lies between the two i's
(if both i's or both j's do then we count both). The number must be 0 or 2, because we have
either i ... i ... j ... j or i ... j ... i ... j or j ... i ... i ... j or i ... j ... j ... i or j ... i ... j ... i or j ... j ...
i ... i. So the sum over all such pairs is even. But the sum is also 1 + 2 + ... + (n-1) = (n-1)n/2
less the number of pairs which straddle the joker. If n is even, then an even number of pairs
must straddle the joker and if n is odd an odd number of pairs. So if n = 4m+1, then the sum is
even less odd = odd, and so 4m+1 is impossible. Similarly, if n = 4m+2, then the sum is odd
less even = odd, so 4m+2 is impossible. Thus we must have n = 0 or 3 mod 4. In particular, n
= 1, 2, 5, 6, 9, 10 are impossible. We show below that n = 3, 4, 7, 8 are possible.

113J232

1134J3242

1125267J5346374

11262578J65347384

Comment. See also Chinese 86/B2. This is a variant on Langford's problem (which is the
same except that there is no joker and we require k numbers between the two k's (not k-1).

25th CanMO 1993

Problem 1
Show that there is a unique triangle such that (1) the sides and an altitude have lengths with
are 4 consecutive integers, and (2) the foot of the altitude is an integral distance from each
vertex.

Solution
Answer: sides AB = 15, BC = 14, CA = 13, altitude AD = 12, BD = 9, DC= 5.
We need the familiar result that the sides of any right-angled triangle with integral sides may
be written as (m2+n2)d, (m2-n2)d, 2mnd for integers m, n, d (see below).
Let the triangle be ABC, with altitude AD. AD differs in length from AB and AC by either 1,
2 or 3. So we have to find two right-angled triangles where the hypoteneuse differs from
another side by 1, 2 or 3. Suppose AB = (m2+n2)d and AD =(m2-n2)d. If AB - AD = 1, then d
= 1 and 2n2 = 1, which is impossible. If AB - AD = 2, then we must have d = 1 and n = 1, so
AB = m2+1, AD = m2 - 1 and BD = 2m. Similarly, AB - AD = 3 is impossible.
If AD has the other form 2mnd, then AC - AD = 1 implies d = 1 and m = n+1, so AC =
2n2+2n+1, AD = 2n2+2n, CD = 2n+1. Similarly, AC - AD = 2 is impossible, and AC - AD = 3
implies d = 3 and m = n+1, so AC = 6n2+6n+3, AD = 6n2+6n, CD = 6n+3.
So we have to consider three possibilities. Either (1) AB = m2+1, AD = m2-1 = 2n2+2n, AC =
m2, BC = 2m+2n+1 = m2±2, or (2) AB = m2+1, AD = m2-1 = 6n2+6n, AC = m2+2, BC =
2m+6n+3 = m2, or (3) AB = 2m2+2m+1, AD = 2m2+2m = 6n2+6n, AC = 6n2+6n+1, BC =
2m+1+6n+3 = 6n2+6n+2.
In case (1) we have from the relation for AC, 2n+1 = m2-2n, so the relation for BC gives
2m+m2-2n = m2±2, so m = n±1 and n4 ±2n2= 2n2 + 2n giving n3-4n-2 = 0 or n3 = 2, which have
no integral solutions.
In case (2) we have similarly m = 3n2-1, 3n3-4n-2 = 0, which has no integral solutions.
In case (3) we have m = 3n2-1 and 3n3-2n-1 = 0, which has the integral solution n = 1 and
hence m = 2, giving the solution shown above.
The result about right-angled triangles can be proved as follows. Assume a, b, c are the sides
of such a triangle with a2 = b2 + c2. Take out the common factor d, so that a = Ad, b = Bd, c =
Cd, with A, B, C having no common factor. We can assume that one of A and C are both odd.
For if both are even, then B would also be even, contradicting our assumption that the three
numbers have no common factor. If A is odd and C is even, then B must be odd and we can
interchange B and C. Finally, we cannot have A even and B odd, for then C is odd, and hence
A2 is 2 mod 4, but a square cannot be 2 mod 4.
So A + C and A - C have a common factor 2. They cannot have a common factor 4, for then
A and C would not be odd. But (A + C)(A - C) = B2, so we must have A + C = 2m2, A - C =
2n2 for some relatively prime m, n. Hence A = m2 + n2, C = m2 - n2, B = 2mn and a = (m2 +
n2)d, B = 2mnd, C = (m2 - n2)d, as claimed.

Problem 2

Show that the real number k is rational iff the sequence k, k + 1, k + 2, k + 3, ... contains three
(distinct) terms which form a geometric progression.

Solution

Suppose there are three such terms k + a, k + b, k + c. Then (k + b)2 = (k + a)(k + c), so k(2b -
a - c) = ac - b2. If 2b - a - c = 0, then also ac - b2 = 0, so b is both the AM and the GM of a and
c. Hence a = c. But a, b, c are assumed to be unequal, so 2b - a - c is non-zero, hence k = (ac -
b2)/(2b - a - c), which shows that k is rational.

Conversely, if k = m/n with m and n non-zero, then kmn = m2, so (k + m)2 = k(k + mn + 2m),
which shows that the three terms k, k + m and k + mn + 2m are a GM of distinct terms.
Finally if k = 0, the terms 1, 2 and 4 are a GM.

Problem 3

The medians from two vertices of a triangle are perpendicular, show that the sum of the
cotangent of the angles at those vertices is at least 2/3.
Solution

Let the triangle be ABC with medians AM and BN meeting at G. Angle A = angle NAG +
angle BAG. Without loss of generality we may take AG = 2, GM = 1, GN = x, BG = 2x. So
cot A = (cot NAG cot BAG - 1)/(cot NAG + cot BAG) = ( (2/x) (1/x) - 1)/(2/x + 1/x) = (2 -
x2)/(3x). Similarly cot B = cot(ABG + MBG) = (2x.x - 1)/(2x + x). So cot A + cot B = (x2 +
1)/(3x), which is at least 2/3 with equality iff x = 1.

Problem 4

Several schools took part in a tournament. Each player played one match against each player
from a different school and did not play anyone from the same school. The total number of
boys taking part differed from the total number of girls by 1. The total number of matches
with both players of the same sex differed by at most one from the total number of matches
with players of opposite sex. What is the largest number of schools that could have sent an
odd number of players to the tournament?

Solution

Answer: 3.

Let there be n schools. Suppose the ith school sends Bi boys and Gi girls. Let B = ∑ Bi and G
= ∑ Gi. We are given that |B - G| = 1.

The number of same sex matches is 1/2 ∑ Bi(B - Bi) + 1/2 ∑ Gi(G - Gi) = (B2 - ∑ Bi2 + G2 - ∑
Gi2). The number of opposite sex matches is ∑ Bi(G - Gi) = BG - ∑ BiGi. Thus we are given
that B2 - ∑ Bi2 + G2 - ∑ Gi2 - 2BG + 2 ∑ BiGi = 0 or ±2. Hence (B - G)2 - ∑(Bi - Gi)2 = 0 or ±2.
But (B - G)2 = 1, so ∑(Bi - Gi)2 = -1, 1 or 3. It cannot be negative, so it must be 1 or 3. Hence
Bi = Gi except for 1 or 3 values of i, where |Bi - Gi| = 1. Thus the largest number of schools
that can have Bi + Gi odd is 3.

Problem 5

A sequence of positive integers a1, a2, a3, ... is defined as follows. a1 = 1, a2 = 3, a3 = 2, a4n =
2a2n, a4n+1 = 2a2n + 1, a4n+2 = 2a2n+1 + 1, a4n+3 = 2a2n+1. Show that the sequence is a permutation of
the positive integers.

Solution

The first few terms are: a1 = 1, a2 = 3, a3 = 2, a4 = 6, a5 = 7, a6 = 5, a7 = 4.


We claim that the terms am for m = 2n, 2n+1, 2n+2, ... , 2n+1-1 are a permutation of 2n, 2n+1,
2n+2, ... , 2n+1-1. By inspection this is true for n = 0, 1 and 2. Suppose it is true for n. Let S =
{2n, 2n+1, ... , 2n+1-1} and S' = {2n+1, 2n+1+1, ... , 2n+2}. As m runs through the numbers in S'
which are 0 or 3 mod 4, [m/2] runs through the numbers in S and conversely as m runs
through the even numbers in S, 2m runs through the numbers in S' which are 0 mod 4, and as
m runs through the odd numbers in S, 2m+1 runs through the numbers in S' which are 3 mod
4. Thus the set {am: m belongs to S' and is 0 or 3 mod 4} = {k : k belongs to S' and is even}.

The odd numbers in S' are found by increasing all the even numbers in S' by 1. But a4n+1 =
a4n+1 and a4n+2 = a4n+3+2, so the set {am: m belongs to S' and is 1 or 2 mod 4} is formed by
increasing by 1 the members of the set {am: m belongs to S' and is 0 or 3 mod 4}. Hence {am:
m belongs to S' and is 1 or 2 mod 4} = {k : k belongs to S' and is odd}. So we have
established the result for n+1.

26th CanMO 1994

Problem 1
Find -3/1! + 7/2! - 13/3! + 21/4! - 31/5! + ... + (19942 + 1994 + 1)/1994!

Solution
∑ (-1)n (n/n! + 1/n!) = -1 + 1/1994! (the series telescopes). Similarly, ∑ (-1)n n2/n! = ∑ (-1)n
n/(n-1)! = ∑ (-1)n (1/(n-2)! + 1/(n-1)! ) = 1/1993! . So expression given is -1 + 1/1994! +
1/1993! = -1 + 1995/1994! .
Problem 2

Show that every power of (√2 - 1) can be written in the form √(k+1) - √k.

Solution

Put a = (√2 - 1)n, b = (√2 + 1)n. Put c = (b + a)/2, d = (b - a)/2, so that a = c - d. Expanding by
the binomial theorem, we see that for n even, c and d/√2 are both integers, and for n odd, c/√2
and d are integers. So for any n, c2 and d2 are integers. But c2 - d2 = ab = 1. Thus putting k =
d2, we have a = √(k+1) - √k.

Comment. There are many different approaches to this problem, all of which work!

Problem 3
25 people sit in circle. They vote for or against an issue every hour. Each person changes his
vote iff his vote was different from both his neighbours on the previous vote. Show that after
a while no one's vote changes.

Solution

Let Sn be the set of people who do not change their votes at round n (for n > 1). Obviously Sn
is a subset of Sn+1. Suppose Sn is non-empty but does not include everyone. Then we can find
two adjacent people x in Sn and y not in Sn. Since y voted differently in round n, y must have
voted the opposite way to x in round n-1. But x votes the same way in round n, so y must vote
the same way as x in round n. Hence y belongs to Sn+1. So if S2 is non-empty then Sn keeps on
growing until it includes everyone.

That is true for any n. But if n is even, then it is possible to have S2 empty. As you move
around the circle people vote alternately for and against. If n is odd, that is impossible.

Problem 4

AB is the diameter of a circle. C is a point not on the line AB. The line AC cuts the circle
again at X and the line BC cuts the circle again at Y. Find cos ACB in terms of CX/CA and
CY/CB.

Solution

Answer: cos2ACB = (CX/CA).(CY/CB). It is the positive root for C outside the circle and the
negative root for C inside the circle.

For C outside the circle, we have cos ACB = CX/CB = CY/CA (one should check that this is
true in all configurations). Hence result.

For C on the circle it is still true because cos ACB = 0 and CX = CY = 0. For C inside the
circle, cos ACB = - cos ACY = - CY/CA and also = - cos BCX = - CX/CB.

Problem 5

ABC is an acute-angled triangle. K is a point inside the triangle on the altitude AD. The line
BK meets AC at Y, and the line CK meets AB at Z. Show that ∠ADY = ∠ADZ.

Solution
Let the rays DZ, DY meet the line through A parallel to BC at U, V respectively. Then UZA
is similar to DZB, so UA/ZA = DB/ZB. Similarly VA/YA = DC/YC. Hence UA/VA = ZA
(DB/ZB) (YC/DC) 1/YA = (DB/DC) (YC/YA) (ZA/ZB). But that is 1 by Ceva's theorem.
Hence UA = VA, so ∠ADZ = ∠ADY.

27th CanMO 1995

Problem 1
Find g(1/1996) + g(2/1996) + g(3/1996) + ... + g(1995/1996) where g(x) = 9x/(3 + 9x).

Solution
g(k) + g(1-k) = 1, so the sum is 997 + g(1/2) = 997 1/2.

Problem 2

Show that xxyyzz ≥ (xyz)(x+y+z)/3 for positive reals x, y, z.

Solution

Assume x ≥ y ≥ z. Then x/z, x/y, y/z are all at least one and (x-z)/3, (x-y)/3, (y-z)/3 are all
positive. Hence (x/z)(x-z)/3, (x/y)(x-y)/3, (y/z)(y-z)/3 are all at least one. Hence their product is at
least 1, which is the required relation.

Problem 3

A convex n-gon is divided into m quadrilaterals. Show that at most m - n/2 + 1 of the
quadrilaterals have an angle exceeding 180 degrees.

Solution

Suppose there are k vertices inside the n-gon. Then we have a total of n + k vertices, m + 1
faces and (4m + n)/2 edges. So using V + F = E + 2, we have n + k + m + 1 = 2m + n/2 + 1.
Hence k = m - n/2 + 1. A quadrilateral can have at most one angle > 180o. The vertex with
that angle must be inside the n-gon (which is convex), so there are at most k quadrilaterals
with an angle > 180o.
Problem 4

Show that for any n > 0 and k ≥ 0 we can find infinitely many solutions in positive integers to
x13 + x23 + ... + xn3 = y3k+2.

Solution

It is sufficient to consider k = 0, for if N2 is a sum of n positive cubes, then multiplying


through by N3k gives N3k+2 as a sum of n positive cubes.

Moreover, it is sufficient to find a single example of N2 as a sum of n positive cubes, for then
multiplying through by any m6 gives a larger example.

But we have the familiar formula 13 + 23 + ... + n3 = ( n(n+1)/2 )2.

Problem 5

0 < k < 1 is a real number. Define f: [0, 1] → [0, 1] by f(x) = 0 for x ≤ k, 1 - (√(kx) + √( (1-k)
(1-x) ) )2 for x > k. Show that the sequence 1, f(1), f( f(1) ), f( f( f(1) ) ), ... eventually becomes
zero.

Solution

We have f(1) = 1 - k. Also for x > k, f(x) = k + x - 2kx - 2 √(kx(1-k)(1-x) ). We have x > k, so
(1-k) > (1-x) and kx(1-k)(1-x) > k2(1-x)2 . Hence f(x) < k + x - 2kx - 2k(1-x) = x - k. So if x >
k, then applying f reduces x by at least k. So applying it n times either reduces x below k or
reduces it by at least nk. But k > 0, so applying f sufficiently many times must reduce x below
k and thereafter it gives 0.

28th CanMO 1996

Problem 1
The roots of x3 - x - 1 = 0 are r, s, t. Find (1 + r)/(1 - r) + (1 + s)/(1 - s) + (1 + t)/(1 - t).

Solution
Put y = (1 + x)/(1 - x). Then x = (y - 1)/(y + 1), so y satisfies (y - 1)3 - (y - 1)(y + 1)2 - (y + 1)3
= 0 or y3 + 7y2 - y + 1 = 0. So the sum of the roots is -7.
Problem 2

Find all real solutions to the equations x = 4z2/(1 + 4z2), y = 4x2/(1 + 4x2), z = 4y2/(1 + 4y2).

Solution

It is immediate from the equations given that 0 <= x, y, z < 1. Rearranging, we have 4z2 = x/(1
- x), 4x2 = y/(1 - y), 4y2 = z/(1 - z). Multiplying gives (xyz)2(1 - x)(1 - y)(1 - z) = xyz/64. If x
= 0, then from the first equation in the question, z = 0 and hence also y = 0. That is one
solution. Otherwise we have xyz non-zero and hence x(1 - x) y(1 - y) z(1 - z) = 1/64. But x(1 -
x) = 1/4 - (x - 1/2)2 <= 1/4 with equality iff x = 1/2. Similarly for y and z, so the only other
solution is x = y = z = 1/2.

Problem 3

Let N be the number of permutations of 1, 2, 3, ... , 1996 in which 1 is fixed and each number
differs from its neighbours by at most 2. Is N divisible by 3?

Solution

Let the number of permutations of 1, 2, ... , n with 1 fixed and each number differing from its
neighbours by at most 2 be p(n). The first number must be 1, so the second number must be 2
or 3. If the second number is 2, then the number of permutations of 2, 3, ... , n with 2 fixed
and each number differing from its neighbours by at most 2 is p(n-1). If the second number is
3, then the third number is 2, 4 or 5. If it is 2, then the fourth number must be 4. In that case,
the number of permutations of 4, 5, ... , n with 4 fixed etc is p(n-3). So suppose it is not 2.

If we get two adjacent numbers (m m+1 or m m-1) then either all following numbers are
greater or all following numbers are less. In the sequence 1, 3, ... we have missed out a
smaller number, namely 2, so after the first even number we must go back to it. Hence after
the first even number we cannot have any larger numbers. In other words, the only possibility
for 1, 3, (not 2), ... is all the odd numbers in increasing sequence followed by all the even
numbers in decreasing sequence, eg 1, 3, 5, 7, 9, 8, 6, 4, 2.

Thus we have established that p(n) = p(n-1) + p(n-3) + 1. Checking the first few values, we
find p(1) = 1, p(2) = 1, p(3) = 2, p(4) = 4, p(5) = 6, p(6) = 9. So mod 3, we have:

n 1 2 3 4 5 6 7 8 9 10 11 12 13 14 15 16 17

p(n) 1 1 2 1 0 0 2 0 1 1 2 1 0 0 2 0 1

So there is a cycle of length 8. 1996 = 4 mod 8, so p(1996) = p(4) = 1 mod 3.

Problem 4
In the triangle ABC, AB = AC and the bisector of angle B meets AC at E. If BC = BE + EA
find angle A.

Solution

Take X on BC with BX = BE. Then CX = AE and so CX/CE = AE/CE = AB/BC = AC/BC.


The the triangles BAC and CXE have a common angle C and the ratio of the two sides
containing the angle the same. Hence they are similar. So XC = XE. Hence ∠BXE = 2 ∠C.
But ∠C = 90o - A/2 and ∠BXE = 90o - B/4 (since BEX is isosceles) = 90o - (90o - A/2)/4.
Solving gives A = 100o.

Problem 5

Let x1, x2, ... , xm be positive rationals with sum 1. What is the maximum and minimum value
of n - [n x1] - [n x2] - ... - [n xm] for positive integers n?

Solution

[n xi] ≤ n xi and n(x1 + ... + xm) = n, so clearly the value cannot be less than 0. But we can
achieve 0 by taking n to be (for example) the product of the denominators of the xi. So the
minimum value is 0.

If we take n to be one less than the product of the denominators, then every term has its
absolute value reduced by 1, so we get a total value of m-1. But each of (n xi - [n xi]) is
strictly less than one, so n - [n x1] - ... - [n xm] is less than m. It is an integer, so it is at most m
- 1. Hence m - 1 is the maximum value.

29th CanMO 1997

Problem 1
How many pairs of positive integers have greatest common divisor 5! and least common
multiple 50! ?

Solution
Answer: 214.
Let m, n satisfy the conditions. Clearly each must be divisible by 5!. Now 50!/5! is divisible
by 15 distinct primes: 2, 3, 5, 7, 11, 13, 17, 19, 23, 29, 31, 37, 41, 43 and 47. The precise
power in each case does not matter. Suppose 11a is the highest power of 11 dividing 50!/5!.
Then mn must be divisible by 11a. But we cannot have 11 dividing both m and n, otherwise
we would increase their greatest common divisor. So either 11a divides m, or it divides n. To
avoid double-counting, we may assume that 47 divides m. Then for each of the other 14
primes we have two choices, giving 214 possible numbers. [We cannot introduce any
additional factors beyond those in 50!/5! or we would increase the lcm.]

Problem 2

A finite number of closed intervals of length 1 cover the interval [0, 50]. Show that we can
find a subset of at least 25 intervals with every pair disjoint.

Solution

There must be an interval with left-hand endpoint belonging to [n, n+1) for n = 0, 1, 2, ... , 49.
For otherwise the collection would not cover some n+1-ε. Take such an interval for n = 0, 2,
4, ... , 48. That gives 25 disjoint intervals.

Problem 3

Show that 1/44 > (1/2)(3/4)(5/6) ... (1997/1998) > 1/1999.

Solution

Put k = (1/2)(3/4)(5/6) ... (1997/1998). We have 1/2 < 2/3, 3/4 < 4/5 etc, so k < (2/3)(4/5)(6/7)
... (1998/1999). Hence k2 < 1/1999 (the product telescopes). But 442 = 1936 < 1999, so 1/1999
< 1/442. Hence k < 1/44.

1/2 > 1/3, 3/4 > 3/5, 5/6 > 5/7 etc. So k > (1/3)(3/5) ... (1997/1999) = 1/1999.

Problem 4

Two opposite sides of a parallelogram subtend supplementary angles at a point inside the
parallelogram. Show that the line joining the point to a vertex subtends equal angles at the two
adjacent vertices.
Solution

Let the parallelogram be ABCD and the point inside be X. Assume that ∠AXB + ∠CXD =
180o. Translate the parallelogram a distance AD along the line AD so that D moves to A, C
moves to B, A moves to A', B moves to B' and X moves to X'. Then O'AOB has opposite
angles summing to 180o, so it is cyclic. So ∠OAB = ∠OO'B. By construction OO' is parallel
to CDD', so ∠OO'B = ∠B'BO' = ∠BCO, so BO subtends the same angles at A and C.
Similarly, ∠OBC = ∠O'OB = ∠O'AB = ∠ODC, so OC subtends the same angles at B and D.

Problem 5

Find ∑ (-1)k nCk /(k3 + 9k2 + 26k + 24), where the sum is taken from k = 0 to n and nCk is the
binomial coefficient n!/( k! (n-k)! ).

Solution

We note first that ∑ (-1)k nCk = (1 - 1)n = 0 and ∑ (-1)k k nCk = 0. The latter is less obvious,
but ∑ (-1)k k nCk = ∑1n (-1)k n!/( (k-1)! (n-k)! ) = -n ∑ (-1)h (n-1)! /( h! (n-1-h)! ) = -n (1 - 1)n-1
= 0.

Now k3 + 9k2 + 26k + 24 = (k + 2)(k + 3)(k + 4), so if the required sum is s, then s/( (n+1)
(n+2)(n+3)(n+4) ) = ∑0n (-1)k (k + 1) (n+4)C(k+4) = ∑4n+4 (-1)k (k-3) (n+4)Ck. Now we
have shown above that the sum from 0 to n+4 is zero. So s/( (n+1)(n+2)(n+3)(n+4) ) + terms k
= 0, 1, 2, 3 is zero. The k = 0 term is -3, the k = 1 term is 2n+8, the k = 2 term is -(n2 + 7n +
12)/2, and the k = 3 term is zero. Hence s/( (n+1)(n+2)(n+3)(n+4) ) = (n+1)(n+2)/2, so s =
1/(2(n+3)(n+4) ).

Problem 1

How many real x satisfy x = [x/2] + [x/3] + [x/5]?

Solution

Answer: 30.

Put x = 30q + r, where q is an integer and 0 ≤ r < 30. Then x - [x/2] - [x/3] - [x/5] = -q + r -
[r/2] - [r/3] - [r/5]. So x = [x/2] + [x/3] + [x/5] iff q = r - [r/2] - [r/3] - [r/5].

q, [r/2], [r/3] and [r/5] are all integers, so r must be an integer. There are 30 possible integral
values of r (namely 0, 1, 2, ... , 29), so there is one solution x for each.
Problem 2

Find all real x equal to √(x - 1/x) + √(1 - 1/x).

Solution

Squaring are rearranging: x3 - x2 - x - 2 = 2√(x3 - x2 - x + 1). Squaring again, x2(x4 - 2x3 - x2 -


2x + 1) = 0. Referring to the original equation, x = 0 is not a solution, so we must have x4 -
2x3 - x2 - 2x + 1 = 0. Factorising (x2 - x - 1)2 = 0, so x = (1 + √5)/2 or (1 - √5)/2. But referring
to the original equation, we must have x ≥ 1, so the only candidate is x = (1 + √5)/2.

With this value 1/x = (√5 - 1)/2, so x - 1/x = 1 and 1 - 1/x = ( (√5 - 1)/2)2 and hence √(x - 1/x)
+ √(1 - 1/x) = x, as required.

Problem 3

Show that if n > 1 is an integer then (1 + 1/3 + 1/5 + ... + 1/(2n-1) )/(n+1) > (1/2 + 1/4 + ... +
1/2n)/n.

Solution

We have 1/2 + 1/3 + 1/5 + ... + 1/(2n-1) > 1/2 + 1/4 + 1/6 + ... + 1/2n). Also 1/2 + 1/2 + ... +
1/2 ≥ 1/2 + 1/4 + ... + 1/(2n), so 1/2 ≥ (1/2 + 1/4 + ... + 1/(2n) )/n. Adding, 1 + 1/3 + 1/5 + ... +
1/(2n-1) > (1/2 + 1/4 + ... + 1/(2n) )(1 + 1/n).

Problem 4

The triangle ABC has ∠A = 40o and ∠B = 60o. X is a point inside the triangle such that
∠XBA = 20o and ∠XCA = 10o. Show that AX is perpendicular to BC.

Solution

We use Ceva's theorem. (sin BAX/sin CAX) (sin ACX/sin BCX) (sin CBX/sin ABX) = 1. So,
putting ∠BAX = x, we have sin x sin 10o sin 40o = sin(40o-x) sin 70o sin 20o. But sin 40o = 2
sin 20o cos 20o = 2 sin 20o sin 70o, so 2 sin x sin 10o = sin(40o-x). Putting x = 30o + y, we get 2
sin(30o+y) sin 10o = sin(10o-y). Expanding: cos y sin 10o + √3 sin y sin 10o = sin 10o cos y -
cos 10o sin y, so sin y(cos 10o + √3 cos 10o) = 0. Hence sin y = 0, so y = 0o and x = 30o. So AX
is perpendicular to BC (the angle between AX and BC is 180o - 30o - 60o = 90o).
Problem 5

Show that non-negative integers a ≤ b satisfy (a2 + b2) = n2(ab + 1), where n is a positive
integer, iff they are consecutive terms in the sequence ak defined by a0 = 0, a1 = n, ak+1 = n2ak -
ak-1.

Solution

If n = 1, then the sequence is 0, 1, 1, 0, -1, -1, 0, 1, 1, ... . Thus the only consecutive non-
negative terms a, b with a ≤ b are 0, 1 and 1, 1 both of which satisfy the equation.

Conversely suppose that a ≤ b is a solution for n = 1. Then a2 + b2 = ab + 1. If 1 < a, then 1 <


a2, ab ≤ b2, so 1 + ab < a2 + b2. Contradiction. So a = 0 or 1. If a = 0, then b2 = 1, so b = 1. If a
= 1, then 1 + b2 = b + 1, so b = 0 or 1, but b ≥ a = 1, so b = 1. Thus the only solutions are a =
0, b = 1, or a = 1, b = 1.

So assume n > 1. It is a trivial induction to show that ak < ak+1. Now it is an easy induction on
k to show that consecutive terms ak-1, ak satisfy the equation. It is true for k = 1: (02 + n2) =
n2(0.n + 1). Suppose it is true for k. Then we have ak+1 + ak-1 = n2ak. Hence ak+12 - ak-12 =
n2ak(ak+1 - ak-1). Adding to ak2 + ak-12 =n2(akak-1 + 1), we get ak+12 + ak2 =n2(ak+1ak + 1), which
completes the induction.

Now suppose that a ≤ b is any solution in non-negative integers of a2 + b2 = n2(ab + 1). The
idea is to show that n2a - b, a is a smaller solution.

If a = b, then 2a2 = n2(a2 + 1) ≥ 4(a2 + 1) > 2a2. Contradiction. So a < b. If a = 0, then b2 = n2,
so b = n. This solution belongs to the sequence. So assume a > 0.

If b > n2a, then b >= n2a + 1, so b2 ≥ n2ab + b > n2ab + n2a ≥ n2(ab + 1), so a2 + b2 > n2(ab + 1).
Contradiction. So n2a - b ≥ 0. If n2a ≥ a + b, then n2ab ≥ ab + b2 > a2 + b2. Contradiction. So
n2a - b < a. Finally, (n2a - b)2 + a2 = n4a2 - 2n2ab + a2 + b2 = n2( a(n2a - b) + 1) + (a2 + b2 - n2(ab
+ 1) ), so if a, b is a solution, then so is n2a - b, a.

Thus if we start with any solution 0 < a ≤ b, we can derive a solution a' < a, where the
relationship between a', a and b is the same as that between ak-1, ak and ak+1. This process must
terminate, so eventually we get a solution 0, c. But we have shown that this must be 0, n. So
we must have been moving down the sequence ak. Hence a, b must be consecutive terms in
that sequence.

31st CanMO 1999


Problem 1
Find all real solutions to the equation 4x2 - 40[x] + 51 = 0.

Solution
Answer: 2x = √29, √189, √229 or √269.
To orient ourselves notice that 4x2 - 40x + 51 = (2x - 17)(2x - 3), so we expect roots to lie in
the range 1 to 9.
Let f(x) = 4x2 - 40[x] + 51. For x <= 1 f(x) > 51, so there are no solutions with x < 1.
f(1) = 15, f(2-) = 16 - 40 + 51 = 27, so there are no solutions in the interval [1, 2).
f(2) = -13, f(3-) = 7, so we expect a solution in [2, 3). For all x in this interval we have f(x) =
4x2 - 29, so the unique solution is 2x = √29.
f(3) = -33, f(4-) = -5, so there are no solutions in [3, 4).
f(4) = -45, f(5-) = -9, so there are no solutions in [4, 5).
f(5) = -49, f(6-) = -5, so there are no solutions in [5, 6).
f(6) = -45, f(7-) = 7, so we expect a solution in [6, 7). For all x in this interval we have f(x) =
4x2 - 189, so the unique solution is 2x = √189.
f(7) = -33, f(8-) = 27. For all x in the interval [7, 8) we have f(x) = 4x2 - 229, so the unique
solution is 2x = √229.
f(8) = -13, f(9-) = 55. For all x in the interval [8, 9) we have f(x) = 4x2 - 269, so the unique
solution is 2x = √269.
f(9) = 15. f(10-) = 91, so there are no solutions in [9, 10). For x >= 10, we have 4x2 - 40[x] +
51 ≥ 4x2 - 40x + 51 = 4x(x - 10) + 51 > 0, so there are no solutions.

Problem 2

ABC is equilateral. A circle with center on the line through A parallel to BC touches the
segment BC. Show that the length of arc of the circle inside ABC is independent of the
position of the circle.

Solution

Let the circle meet the segment AC at X and the line AB at Y with Y outside the segment AB.
Let O be the center of the circle. Then ∠OAY = ∠OAC = 60o. But the line OA is a diameter
of the circle, so XY must be perpendicular to AO and hence ∠AYX = 30o. So the arc inside
the circle subtends the constant angle 30o at the circumference, so it must have constant
length.

Problem 3

Find all positive integers which equal the square of their number of positive divisors.

Solution
Let N = paqb ... . Then the square of the number of positive divisors of N is (a + 1)2(b + 1)2 ... .
Any square has all its prime factors to an even power, so a, b, ... must all be even. So writing a
= 2m, b = 2n etc, we have pmqn ... = (2m+1)(2n+1) ... . Since the rhs is odd, all the primes p, q,
... must be odd.

A trivial induction shows that 2n + 1 < 3n for n > 1. Hence 2n + 1 < pn for n > 1 for any odd
prime p. For n = 1, 2n + 1 = 3 < p unless p = 3. So 2n+1 < pn, where p is an odd prime and n is
a positive integer, except in the case n = 1, p = 3.

Thus we can have at most one prime p in the factorisation of N and N = 1 or 32.

Problem 4

X is a subset of eight elements of {1, 2, 3, ... , 17}. Show that there are three pairs of (distinct)
elements with the same difference.

Solution

Let the elements of X be a1 < a2 < ... < a8. The 7 differences a2 - a1, a3 - a2, ... a8 - a7 have sum
a8 - a1. The 6 differences a3 - a1, a4 - a2, ... , a8 - a6 have sum a8 + a7 - a1 - a2. So the 13
differences together have sum 2a8 + a7 - a2 - 2a1 ≤ 34 + 16 - 2 - 2 = 46. But if no difference
occurs more than twice then the sum must be at least 2(1 + 2 + 3 + 4 + 5 + 6) + 7 = 49.

Problem 5

x, y, z are non-negative reals with sum 1, show that x2y + y2z + z2x ≤ 4/27. When do we have
equality?

Solution

Assume x ≥ y, z. We have (x + z/2)2(y + z/2) - x2y - y2z - z2x = (x - y)yz + xz(x - z)/2 + yz2/4
+ z3/8 > 0, unless z = 0. So if z > 0, we get a larger sum for x' = x+z/2, y' = y+z/2, z' = 0. So
we can assume z = 0.

By AM/GM applied to x, x, 2y, we have 2x2y ≤ ( (2x + 2y)/3)3 = 8/27 and hence x2y ≤ 4/27
with equality iff x = 2/3, y = 1/3. Thus the original inequality holds with equality iff (x, y, z) =
(2/3, 1/3, 0), (0, 2/3, 1/3) or (1/3, 0, 2/3).

32nd CanMO 1980


Problem 1
Three runners start together and run around a track length 3L at different constant speeds, not
necessarily in the same direction (so, for example, they may all run clockwise, or one may run
clockwise). Show that there is a moment when any given runner is a distance L or more from
both the other runners (where distance is measured around the track in the shorter direction).

Solution
Let the runners be A, B, C. Using a rotating frame, if necessary, we may take A to be at rest.
wlog B is faster than C. Take the first time when C reaches a point L from the start. If B is not
more than twice as fast as C, then B will also be a distance at least L from the start (whichever
way B runs). If B runs more than twice as fast as C, then whilst C runs the next distance L
around the track, C is always at least L from A and B runs a distance of at least 2L. At some
point during this period B must also be a distance at least L from A.

Problem 2

How many permutations of 1901, 1902, 1903, ... , 2000 are such that none of the sums of the
first n permuted numbers is divisible by 3 (for n = 1, 2, 3, ... , 2000)?

Solution

There are 34 numbers equal to 2 mod 3, and 33 each equal to 0 and 1 mod 3. The multiples of
3 do not affect the values mod 3, so consider the sequence of the other terms. Call terms equal
to 1 mod 3, 1-terms and terms equal to 2 mod 3, 2-terms. If we start with a 1-term, then the
next term must be a 1-term. The sum is then 2 mod 3, so the next term must be a 2-term, the
sum is then 1 mod 3 and so the next term must be a 1-term, and so on. So after the first two
terms, we must alternate between 1-terms and 2-terms. Similarly, if we start with a 2-term, the
next term must be a 2-term, but then we must alternate. But there are more 2-terms than 1-
terms, so we cannot start with a 1-term. Thus (ignoring the terms divisible by 3), the sequence
must be 2-term, 2-term, 1-term, 2-term, 1-term, ... , 2-term, 1-term.

Thus we may start by placing the terms divisible by 3 in any positions except the first. That
gives 99!/66! possibilities. The pattern of 2-terms and 1-terms is then determined and so in
each case there are 34! 33! ways or arranging the 2-terms and 1-terms. Thus there are (99! 34!
33!)/66! possibilities in all.

Problem 3
Show that in any sequence of 2000 integers each with absolute value not exceeding 1000 such
that the sequence has sum 1, we can find a subsequence of one or more terms with zero sum.

Solution

Suppose the result is false. We can permute the sequence to a1, a2, ... , a2000 so that an has
opposite sign to sn-1 = a1 + a2 + ... + an-1 for n > 1. This is easily proved by induction. Note that
no terms can be zero, or we would have a subsequence of one term with zero sum. Having
chosen a1, ... , an-1 with n < 2000, we the remaining terms have sum 1 - sn-1. We cannot have sn-
1 = 0 or 1 otherwise we have a zero sum subsequence (with a1, ... , an-1 or the remaining terms),
hence the sum of the remaining terms has opposite sign to sn-1. So we can find at least one
term with the opposite sign to sn-1.

Now consider the values assumed by s1, s2, ... , sn. Each value must be different from zero (or
we would have a zero sum subsequence) and only s1 can have the value 1000 or -1000. Thus
there are at most 1999 values available. So two terms sm and sn with m < n must have the same
value. But then am+1 + ... + an = 0. Contradiction.

Problem 4

ABCD is a convex quadrilateral with AB = BC, ∠CBD = 2 ∠ADB, and ∠ABD = 2 ∠CDB.
Show that AD = DC.

Solution

Let the diagonals intersect at E and extend the ray DB to meet the circle center B radius BA at
F. Then ∠BFC = ∠ADB and ∠BFA = ∠BDB. So AFCD is a parallelogram, so its diagonals
bisect each other, so E is the midpoint of AC. But ABC is isosceles, so DF and AC are
perpendicular. Hence AD = DC.

Problem 5

A non-increasing sequence of 100 non-negative reals has the sum of the first two terms at
most 100 and the sum of the remaining terms at most 100. What is the largest possible value
for the sum of the squares of the terms?

Solution

Let the sequence by x1, x2, ... , x100. Given any sequence satisfying the conditions with the sum
of the first two terms less than 100, we can obtain another sequence which also satisfies the
conditions and has larger sum of squares by increasing the first term. So we may assume that
x1 + x2 = 100. So the sum of squares is at most (100 - x2)2 + x22 + ... + x1002 = 10000 + 2x22 -
200x2 + x32 + ... + x1002 <= 10000 + 2x22 - x2(x1 + x2 + ... + x100) + x32 + ... + x1002 = 10000 +
x2(x2 - x1) + x3(x3 - x2) + ... + x100(x100 - x2).

Hence the maximum value is 10000 and is achieved iff all of x2(x2 - x1), x3(x3 - x2), ... ,
x100(x100 - x2) are zero.

x2(x2 - x1) = 0 implies either x2 = 0 or x2 = x1. The former gives the unique solution x1 = 100,
other terms zero. The latter implies x1 = x2 = 50 and every other term must be 0 or 50, which
gives the unique solution 50, 50, 50, 50, 0, ... , 0.

33rd CanMO 2001

Problem
A quadratic with integral coefficients has two distinct positive integers as roots, the sum of its
coefficients is prime and it takes the value -55 for some integer. Show that one root is 2 and
find the other root.

Solution
Let the roots be m and n and the leading coefficient k, then the quadratic is k(x2 - (m + n)x +
mn). The sum of its coefficients is k(m - 1)(n - 1). We are told this is prime, hence k = 1 and
m = 2. So the quadratic is (x - 2)(x - n) with n > 2 an integer. We have (x - 2)(x - n) = -55 for
some x. But the only factors of 55 are 1, 5, 11, 55, so x = 3, n = 58, or x = 7, n = 18. But n -1
is prime, so n = 18.

Problem 2

The numbers -10, -9, -8, ... , 9, 10 are arranged in a line. A player places a token on the 0 and
throws a fair coin 10 times. For each head the token is moved one place to the left and for
each tail it is moved one place to the left. If we color one or more numbers black and the
remainder white, we find that the chance of the token ending up on a black number is m/n
with m + n = 2001. What is the largest possible total for the black numbers?

Solution

The token must end on an even number, so clearly we color 1, 3, 5, 7, 9 black and -1, -3, -5, -
7, -9 white. Let p(n) be the probability of the token ending up on n. Then p(10) = p(-10) =
1/1024, p(8) = p(-8) = 10/1024, p(6) = p(-6) = 45/1024, p(4) = p(-4) = 120/1024, p(2) = p(-2)
= 210/1024, p(0) = 252/1024.

2001 = 3.23.29, so if m+n = 2001, then m = ka, n = kb, where a < b are coprime and k = 1, 3,
23, 29, 69, 78, 667 or 2001. b must be a power of 2, so the possibilities are: a/b = 0/1, 1/2,
7/16, 13/16, 5/64, 23/64, 155/512, 977/1024. Hence the probability must be h/1024 where h =
0, 512, 448, 832, 80, 368, 310 or 977.

We find that the only possibilities are 0, 512 = 252 + 120 + 120 + 10 + 10, 512 = 210 + 210 +
45 + 45 + 1 + 1, 310 = 210 + 45 + 45 + 10 = 977 = 252 + 210 + 210 + 120 + 120 + 45 + 10 +
10. These give sums (for the even numbers) of 0, 0, 0, 10, 6. So the best is 10, obtained by
coloring black the even numbers 2, 6, -6, 8.

Thus we color -6, 1, 2, 3, 5, 6, 7, 8, 9 black and the rest white for a total of 45, giving a
probability of 310/1024 = 155/512 = 465/1536, where 465 + 1536 = 2001.

Problem 3

The triangle ABC has AB and AC unequal. The angle bisector of A meets the perpendicular
bisector of BC at X. The line joining the feet of the perpendiculars from X to AB and AC
meets BC at D. Find BD/DC.

Solution

Let the perpendiculars from X to the lines AB, AC meet them at Z, Y respectively. Triangles
XBZ, XYC are congruent because XB = XC (X lies on angle bisector), XZ = XY (X lies on
perpendicular bisector) and ∠BZX = ∠CYX = 90o. Hence BZ = CY. Also AZ = AY. By
Ceva's theorem, (AZ/ZB) (BD/DC) (CY/YA) = 1. Hence BD/DC = 1.

Problem 4

A rectangular table has every entry a positive integer. n is a fixed positive integer. A move
consists of either subtracting n from every element in a column or multiplying every element
in a row by n. Find all n such that we can always end up with all zeros whatever the size or
content of the starting table.

Solution

Neither type of move changes an entry mod n-1. So if n > 2, we can never zero entries which
do not start as multiples of n-1. So we need only consider n = 1 and n = 2.

If n = 1, then only the column move achieves anything. But if we have at least two rows and a
column in which not all the elements are equal, then we can never zero the column.

We show that if n = 2, then we can always zero the table. We zero one column at at time. Start
by doubling any row with an odd entry, so that all entries become even. Now suppose the
largest entry in the first column is 2m > 2, we can reduce it by doubling any 2s to 4 and then
subtracting enough 2s to bring the smallest entry back to 2. We must eventually get all entries
in the first column equal to 2. Now subtract 2, to zero the first column. Repeat for the other
columns.

Problem 5

A0, A1, A2 lie on a circle radius 1 and A1A2 is not a diameter. The sequence An is defined by
the statement that An is the circumcenter of An-1An-2An-3. Show that A1, A5, A9, A13, ... are
collinear. Find all A1A2 for which A1A1001/A1001A2001 is the 500th power of an integer.

Solution

Let angle A1A3A2 = x. Then A1A2 = 2A1A3 sin x/2 = 2 sin x/2. We have AnAn-2 = AnAn-1, since
An-2 and An-1 lie on a circle center An. So by symmetry A4 lies on the bisector of ∠A1A3A2, so
∠A4A3A2 = x/2. Hence ∠A4A2A3 = x/2 and ∠A2A4A3 = 180o - x. Similarly A5 lies on the
bisector of ∠A2A4A3, so ∠A5A4A3 = 90o - x/2. Hence ∠A5A3A4 = 90o - x/2 and ∠A3A5A4 = x
= ∠A1A3A2. So triangles A1A3A2 and A3A5A4 are similar.

∠A1A3A5 = ∠A1A3A4 + A4A3A5 = x/2 + 90o - x/2 = 90o. Also, 2A3A4 cos x/2 = A1A3 = 1, so
A3A5/A1A3 = A3A4/A1A2 = 1/(2 sin x). Thus the triangle A5A7A9 is similar to A1A3A5 and
rotated through 180o. Hence A1, A5 and A9 are collinear. Similarly, A4n+1, A4n+5 and A4n+9 are
collinear for any n and hence all of A1, A5, ... , A4n+1, ... are collinear.

Put y = A1A3/A3A5 = 2 sin x. Then A1A1001/A1001A2001 = y500 (because the structure A1 A3 A5 ...
A1001 is similar to A1001A1003 ... A2001 but larger by a factor (y2)250. So we want y to be an
integer. Hence sin x = 1/2 or 1, so x = 30o, 90o or 150o and A1A2 = 2 sin x/2 = 2 sin 15o, 2 sin
45o or 2 sin 75o = (√3 - 1)/√2, √2 or (√3 + 1)/√2.

34th CanMO 2002

Problem 1
What is the largest possible number of elements in a subset of {1, 2, 3, ... , 9} such that the
sum of every pair (of distinct elements) in the subset is different?

Solution
{1, 2, 3, 5, 8} has five elements and all pairs with a different sum. If there is a subset with 6
elements, then it has 15 pairs, each with sum at least 1 + 2 = 3 and at most 8 + 9 = 17. There
are only 15 numbers at least 3 and at most 17, so each of them must be realised. But the only
pair with sum 3 is 1,2 and the only pair with sum 17 is 8, 9 and then 1 + 9 = 2 + 8. So six
elements is impossible.

Problem 2

We say that the positive integer m satisfies condition X if every positive integer less than m is
a sum of distinct divisors of m. Show that if m and n satisfy condition X, then so does mn.

Solution

Suppose k < mn. Then k = am + b, where 0 ≤ a < n, 0 ≤ b < m. We may write a as a sum of
distinct divisors of n. Hence am is a sum of distinct divisors of mn, each of them at least m.
Also b is a sum of distinct divisors of m, each of them less than m. Hence k is a sum of
distinct divisors of mn.

Problem 3

Show that x3/(yz) + y3/(zx) + z3/(xy) ≥ x + y + z for any positive reals x, y, z. When do we
have equality?

Solution

(x4 + y4)/2 ≥ x2y2 with equality iff x = y. Hence x4 + y4 + z4 ≥ x2y2 + y2z2 + z2x2 with equality
iff x = y = z.

(x2y2 + y2z2)/2 ≥ xy2z. Hence x2y2 + y2z2 + z2x2 ≥xyz(x + y + z). So x4 + y4 + z4 ≥ xyz(x + y +
z) with equality iff x = y = z. Dividing by xyz gives the required result.

Problem 4

ABC is an equilateral triangle. C lies inside a circle center O through A and B. X and Y are
points on the circle such that AB = BX and C lies on the chord XY. Show that CY equals AO.
Solution

Let O be the center of the circle. Chasing angles around ABXY we find that triangles AYC
and AOB. Hence YC = OB.

[Let ∠OBA = x. Then ∠ABX = 2x, so ∠XBC = 2x - 60o, so ∠BCX = ∠BXC = 120o - x.
Hence ∠ACY = 180o - 60o - (120o - x) = x. ∠AYC = 180o - ∠ABX = 180o - 2x, so ∠YAC =
x.]

Problem 5

Let X be the set of non-negative integers. Find all functions f: X → X such that x f(y) + y f(x)
= (x + y) f(x2 + y2) for all x, y.

Solution

Putting x = 0 we get y f(0) = y f(y2), so f(y2) = f(0) for all y. That strongly suggests f is
constant. Obviously any constant function satisfies the condition.

Suppose f(x) < f(y) and neither x nor y is zero, then (x + y) f(x) < x f(y) + y f(x) < (x + y) f(y).
Hence f(x) < f(x2 + y2) < f(y). But that is impossible, because we could repeat the argument to
get an infinite number of distinct values between f(x) and f(y). But we know that f(1) = f(0).
Hence f(x) = f(1) for all x > 1. So f is constant.

You might also like